Location via proxy:   [ UP ]  
[Report a bug]   [Manage cookies]                
Download as pdf or txt
Download as pdf or txt
You are on page 1of 261

Over Current Relay: Working Principle and Types

In an over current relay or o/c relay the actuating quantity is only current. There is only one current
operated element in the relay, no voltage coil etc. are required to construct this protective relay.

Over Current Relay- Working Principle Types

In an over current relay or o/c relay the actuating quantity is only current.
There is only one current operated element in the relay, no voltage coil etc. are
required to construct this protective relay.

Working Principle of Over Current Relay

In an over current relay, there would be essentially a current coil. When normal
current flows through this coil, the magnetic effect generated by the coil is not
sufficient to move the moving element of the relay, as in this condition the
restraining force is greater than deflecting force. But when the current through
the coil increased, the magnetic effect increases, and after certain level of
current, the deflecting force generated by the magnetic effect of the coil, crosses
the restraining force, as a result, the moving element starts moving to change
the contact position in the relay.

Although there are different types of over current relays but basic working
principle of over current relay is more or less same for all.

Types of Over Current Relay

Depending upon time of operation, there are various types of OC relays, such
as,

1. Instantaneous over current relay.


2. Definite time over current relay.

3. Inverse time over current relay.

Inverse time over current relay or simply inverse OC relay is again subdivided
as inverse definite minimum time (IDMT), very inverse time, extremely inverse
time over current relay or OC relay.

Instantaneous Over Current Relay

Construction and working principle of instantaneous over current relay


quite simple.
Here generally a magnetic core is wound by current coil. A piece of iron is so
fitted by hinge support and restraining spring in the relay, that when there is
not sufficient current in the coil, the NO contacts remain open. When current in
the coil crosses a present value, the attractive force becomes sufficient to pull
the iron piece towards the magnetic core and consequently the No contacts are
closed.

The preset value of current in the relay coil is referred as pick up setting
current. This relay is referred as instantaneous over current relay, as ideally,
the relay operates as soon as the current in the coil gets higher than pick up
setting current. There is no intentional time delay applied. But there is always
an inherent time delay which can not be avoided practically. In practice the
operating time of an instantaneous relay is of the order of a few milliseconds.
Fig.

Directional Over Current Relays


When fault current can flow in both the directions through the relay, at its location. Therefore, it is
necessary to make the relay respond for a particular defined direction, so that proper discrimination is
possible. This can be achieved by introduction of directional control elements.

Directional Over Current Relays

1. When fault current can flow in both the directions through the relay,
at its location. Therefore, it is necessary to make the relay respond for a
particular defined direction, so that proper discrimination is possible. This can
be achieved by introduction of directional control elements.

2. These are basically power measuring devices in which the system


voltage is used as a reference for establishing the relative phase of the fault
current.

Basically, an AC directional relay can recognize certain difference in phase


angle between two quantities, just as a D.C. directional relay recognize
difference in polarity
The polarizing quantity of a directional relay

1. It is the reference against which the phase angle of the other quantity
is compared. Consequently the phase angle of the polarizing quantity must
remain fixed when other quantity suffers wide change in phase angle.

2. The voltage is chosen as the “polarizing” quantity in the current-


voltage induction type directional relay.

3. Four pole induction cup construction is normally used.

• Prev Page

Distance relay
Principle of operation of distance relay: The basic principle of measurement involves the comparison of
fault current seen by the relay with the voltage at relaying point; by comparing these two quantities.

Distance relay

Distance relay is used for the protection of transmission line & feeders In a
distance relay, instead of comparing the local line current with the current at
far end of line, the relay compares the local current with the local voltage in the
corresponding phase or suitable components of them

Principle of operation of distance relay

1. The basic principle of measurement involves the comparison of fault


current seen by the relay with the voltage at relaying point; by comparing these
two quantities.
2. It is possible to determine whether the impedance of the line up to the
point of fault is greater than or less than the predetermined reach point
impedance

There are two types of torques

Types of distance relay Distance relays are classified depending on their


operating characteristic in the R-X plane

• Impedance Relay

• Mho Relay

• Reactance Relay
Impedance Relay
The torque equation T, for such a relay the current actuates the operating torque and the voltage
actuates the restraining torque, with the usual spring constant K4.

Impedance Relay:

The torque equation T, for such a relay the current actuates the operating
torque and the voltage actuates the restraining torque, with the usual spring
constant K4.

Considering K2 to be negative (as it produces the restraining torque) and


neglecting the torque component due to spring, the equation represents a circle
in the R-X plane.

DISADVANTAGE OF IMPEDANCE RELAY

1. It is not directional.

2. It is affected by the Arc resistance


3. It is highly sensitive to oscillations on the power system, due to large
area covered by its circular characteristic

Differential Relay

Principle of circulating current differential (MERZ-PRIZE) protection, Differential Protection current


balance, Demerits of a Differential Relay( Merz Price Scheme).

Differential Relay

One of the most prevalent and successful method of protecting a circuit is to


arrange relays to compare the currents entering and leaving it, which should be
the same under normal conditions and during an external fault. Any difference
current must be flowing in to a fault within the protected circuit

Principle of circulating current differential (MERZ-PRIZE) protection

The figure below illustrates the principle of differential protection of generator


and transformer, X is the winding of the protected machine. Where there is no
internal fault, the current entering in X is equal in phase and magnitude to
current leaving X. The CT's have such a ratio that during the normal conditions
or for external faults (Through Faults) the secondary current of CT's are equal.
These current say I1 and I2 circulate in the pilot wire. The polarity connections
are such the current I1 and I2 are in the same direction of pilot wire during
normal condition or external faults. Relay operation coil is connected at the
middle of pilot wires. Relay unit is of over current type

During normal condition and external fault the protection system is balanced
and the CT's ratios are such that secondary currents are equal. These current
circulate in pilot wires. The vector differential current I1- I2 which flow through
the relay coil is zero. I1-I2 = 0 (normal condition or external faults) This balance
is disturbed for internal faults. When fault occurs in the protected zone, the
current entering the protected winding is no more equal to the leaving the
winding because some current flows to the fault. The differential I1-I2 flows
through the relay operating coil and the relay operates if the operating torque
is more than the restraining torque. The current I1 and I2 circulate in the
secondary circuit. Hence CT's does not get damaged. Polarities of CT's should
be proper, otherwise the currents I1 and I2 would add up even for normal
condition and mal operate the relay.

Differential Protection current balance

• When this system is applied to electrical equipment (Generator stator


windings, Transformer,

Bus bars etc.) it is called differential current protection.


• When it is applied to lines and cables it is called pilot differential protection
because pilot wires or an equivalent link or channel is required to bring the
current to the relay from the remote end of the line.

The CTs at both ends of the protected circuit connected so that for through load
or through fault conditions current circulates between the interconnected CTs.
The over-current relay is normally connected across equipotential points and
therefore doesn‟t operate.

• Circulating current balance methods are widely used for apparatus protection
where CTs are within the same substation area and interconnecting leads
between CTs are short (e.g. generator stator windings, Transformer, Bus bars
etc.)

• The circulating current balance method is also called longitudinal differential


protection or

Merz-Price differential protection system.

• The current in the differential relay would be proportional to the phasor


difference between the currents that enter and leave the protected circuit. If the
current through the relay exceeds the pick-up value, then the relay will operate.

Demerits of a Differential Relay( Merz Price Scheme)

• Unmatched characteristics of C.T.s :


Though the saturation is avoided, there exist difference in the C.T.
characteristics due to ratio error at high values of short circuit currents. This
causes an appreciable difference in the secondary currents which can operate
the relay. So the relay operates for through external faults.

This difficulty is overcome by using percentage differential relay. In this relay,


the difference in current due to the ratio error exists and flows through relay
coil. But at the same time the average current ( I1 + I2/2) flows through the
restraining coil which produces enough restraining torque. Hence relay
becomes inoperative for the through faults.

• Ratio change due to tap change:

To alter the voltage and current ratios between high voltage and low voltage
sides of a power transformer, a tap changing equipment is used. This is an
important feature of a power transformer. This equipment effectively alters the
turns ratio. This causes unbalance on both sides. To compensate for this effect,
the tapping can be provided on C.T.s also which are to be varied similar to the
main power transformer. But this method is not practicable. The percentage
differential relays ensure relays ensure the stability with respect to the amount
of unbalance occurring at the extremities of the tap change range.

• Difference in lengths of pilot wires:

Due to the difference in lengths of the pilot wires on both sides, the unbalance
condition may result. The difficulty is overcome by connecting the adjustable
resistors in pilot wires on both sides. These are called balancing resistors. With
the help of these resistors, equipotential points on the pilot wires can be
adjusted. In percentage differential relays the taps are provided on the
operating coil and restraining coil to achieve an accurate balance.

• Magnetizing current inrush:


When the transformer is energized, the condition initially is of zero induced
E.m.f. A transient inflow of magnetizing current occurs in to the transformer.
This current is called magnetizing inrush current. This current may be as great
as 10 times the full load current of the transformer. This decays very slowly and
is bound to operate differential protection of the transformer falsely, because
of the temporary difference in magnitude of the primary and secondary
currents.

The factors which affect the magnitude and direction of the magnetizing inrush
current can be one of the following reasons.

a. Size of the transformer.

b. Size of the power system

c. Type of magnetic material used for the core.

d. The amount of residual flux existing before energizing the transformer.

e. The method by which transformer is energized.

If the transformer is energized when the voltage wave is passing through zero,
the magnetizing current inrush is maximum. At this instant, the current and flux
should be maximum in highly inductive circuit. And in a half wave flux reversal
must take place to attain maximum value in the other half cycles. If the residual
flux exists, the required flux may be in same or opposite direction. Due to this
magnetizing current inrush is less or more. If it is more, it is responsible to
saturate the core which further increases its component. This current decays
rapidly for first few cycles and then decays slowly. The time constant L/R of the
circuit is variable as inductance of circuit varies due to the change in
permeability of the core. The losses in the circuit damp the inrush currents.
Depending on the size of the transformer, the time constant of inrush current
varies from 0.2 sec to 1 sec. The waveforms of magnetizing inrush current in
three phases are shown in the figure below.

Static relays
Due to the amplification of energizing signals obtainable, the sources need only provide low power.
Therefore the size of the associated current and voltage transformers could be reduced.

Static relays

Advantages of static relays

• Due to the amplification of energizing signals obtainable, the sources need


only provide low power. Therefore the size of the associated current and
voltage transformers could be reduced.

• Improved accuracy and selectivity.


• Fast operation of relays and hence fast clearance of faults.

• Flexibility of circuitry would allow new and improved characteristics.

• The relays would be unaffected by the number of operations.

Basic circuits employed

• Timers

• Phase comparators

• Amplitude Comparator

• Level detectors

• Integrators

• Polarity detectors

High reliability operational amplifiers are used for realizing the basic
components of static relays.

Over current and earth fault protection


It is customary to have two elements of over current and one element of earth fault protection system in
the most elementary form of protection of three phase feeders.
Over current and earth fault protection

It is customary to have two elements of over current and one element of earth
fault protection system in the most elementary form of protection of three
phase feeders. Different types of feeders employ the over current protection
along with the directional relay so that proper discrimination of an internal
fault is possible. Some examples are illustrated below.

Application of directional relays to parallel feeders

It may be seen from the below given parallel feeders that the relays placed at
the load side of both the lines use directional element which respond to a
direction away from the bus bars. Similarly, the relays placed at the source side
do not require any directional element.

A similar concept of discrimination is also utilized in the below given ring main
feeder and a feeder fed from both the sides. It can be observed that relays
placed near the bus connecting the sources, don not have any directional
feature, where as the rest of the buses, respond to a direction always away from
the source. It is good practice to locate a fault any where among different
sections of the feeders and check whether that particular section only is
isolated without disrupting the power flow in other sections.
Pilot wire schemes for feeder protection

In differential protection scheme, the current entering at one end of the line and
leaving from other end of the line is compared. The pilot wires are used to
connect the relays. Under normal working condition, the two currents at both
ends are equal and pilot wires do not carry any current, keeping relays
inoperative. Under an internal fault condition, the two currents at both the ends
are no longer same, this causes circulating current flow through pilot wires and
makes the relay to trip. The various schemes used with this method of
protection are, 1. Merz-Price Voltage Balance System 2. Translay Scheme

Merz-Price Voltage Balance System

The figure below shows Merz-Price voltage balance system used for the three
phase feeders.

Under normal condition, current entering the line at one end is equal to current
leaving from the other end. Therefore, equal and opposite voltages are induced
in the secondaries of C.T.s. at the two ends resulting in no current flow, through
the relay. Under fault condition, two currents at the two ends are different. Thus
the secondary voltages of both the end C.T.s differ from each other. This
circulates a circulating current through the pilot wires and the relays. Thus the
relays trip the circuit breakers to isolate the faulty section.

The advantages of this method are as follows

1. It can be used for parallel as well as ring main system.

2. It provides instantaneous protection to ground faults.

The limitations of this method are as follows

1. The C.T.s used must match accurately.

2. The pilot wires must be healthy without discontinuity.

3. Economically not suitable as the cost is high due to long pilot wires.

4. Due to long pilot wires, capacitive effects adversely bias the operation of the
relays.

5. The large voltage drop in the pilot wires requiring better insulation.

Translay Scheme

The translay relay is another type of differential relay. The arrangement is


similar to overcurrent relay but the secondary winding is not closed on itself.
Additionally copper ring or copper shading bands are provided on the central
limb as shown in the figure below.

Role of copper ring:

Mainly relays may operate because of unbalance in the current transformers.


The copper rings are so adjusted that the torque due to current induced in the
copper ring due to primary winding of relay is restraining and do not allow the
disc to rotate. It is adjusted just to neutralize the effect of unbalance existing
between the current transformers. The copper rings also neutralize the effect
of pilot capacitive currents. Though the feeder current is same at two ends, a
capacitive current may flow in the pilots. This current leads the secondary
voltage by 90o. The copper rings are adjusted such that no torque is exerted on
the disc, due to such capacitive pilot currents. Therefore in this scheme the
demerits of pilot relaying scheme is somewhat taken care of.

The advantages of this scheme are, 1. Only two pilot wires are required.
2. The cost is very low.

3. The current transformers with normal design can be employed.

4. The capacitive effects of pilot wire currents do not affect the operation of the
relays.

Carrier Current unit protection system

Schematic diagram of the carrier current scheme is shown below. Different


basic components of the same are discussed below. The Coupling capacitor
These coupling capacitors (CU) which offer low reactance to the higher
frequency carrier signal and high reactance to the power frequency signal.
Therefore, it filters out the low ( power) frequency and allows the high
frequency carrier waves to the carrier current equipments. A low inductance is
connected to the CU, to form a resonant circuit.
Wave Traps

The Wave traps ( also known as Line Trap) are inserted between the busbar
and the connection of the CU. These traps are L and C elements connected in
parallel, and they are tuned in such a manner that they offer low reactance to
the power frequency signals and high reactance to the carrier waves. They
ensure that neither of these different frequency signals get mixed up before
being received at the bus bar. Both the CU and the Wave traps are protected
from switching and lightening surges, with the help suitably designed Spark
Gaps or Varistors. Frequency spacing Different frequencies are used in adjacent
lines and the wave traps ensure that carrier signals of other lines do not enter
a particular line section. Therefore, proper choice of frequency bands for
different lines are adopted.

Transmitter Unit

In a Transmitter unit, the carrier frequency in the range of 50 to 500 KHz of


constant magnitude is generated in the oscillator, which is fed to an amplifier.
Amplification is required to overcome any loss in the coupling equipments,
weather conditions, Tee connections in the lines of different size and length.
The amplifier and the oscillators are constantly energized and a connection is
made between the two with the help of a control unit.

The Receiver unit consists of an attenuator and a Band pass filter, which
restricts the acceptance of any unwanted signals. The unit also has matching
transformer to match the line impedance and that of the receiver unit.

• Prev Page

Transformer Protection
The power transformer is one of the most important links in a power transmission and distribution
system.

TRANSFORMER PROTECTION
INTRODUCTION

• The power transformer is one of the most important links in a power


transmission and distribution system.

• It is a highly reliable piece of equipment. This reliability depends on

• adequate design

• careful erection

• proper maintenance

• application of protection system.

PROTECTION EQUIPMENT INCLUDES

1. Surge diverters

2. Gas relay: It gives early warning of a slowly developing fault, permitting


shutdown and repair before severe damage can occur.

3. Electrical relays.

• The choice of suitable protection is also governed by economic


considerations.
Although this factor is not unique to power transformers, it is brought in
prominence by the wide range of transformer ratings used( few KVA to several
hundreds MVA)

• Only the simplest protection such as fuses can be justified for transformers
of lower ratings.

• for large transformers best protection should be provided.

TYPES OF FAULTS AFFECTING POWER TRANSFORMER

• THROUGH FAULTS

a) Overload conditions.

b) External short-circuit conditions.

The transformer must be disconnected when such faults occur only after allowing
a predetermined time during which other protective gears should have operated.

• INTERNAL FAULTS

The primary protection of a power transformer is intended for conditions which


arises as a result of faults inside the protection zone.

1. Phase-to-earth fault or phase- to- phase fault on HV and LV external


terminals
2. Phase-to-earth fault or phase-to- phase fault on HV and LV windings.

3. Interturn faults of HV and LV windings.

4. Earth fault on tertiary winding, or short circuit between turns of a tertiary


windings.

5. So called „incipient‟ faults which are initially minor faults, causing gradually
developing fault. These types of faults are not easily detectable at the winding
terminals by unbalance current or voltage.

NATURE & EFFECT OF TRANSFRMER FAULTS

A faults on transformer winding is controlled in magnitude by

a) Source & neutral earthing impedance

b) Leakage reactance of the transformer

c) Position of the fault on the winding.

Following distinct cases are examined below (1) Star connected winding with
neutral point earthed through an impedance

Earth fault on resistance earthed star winding


Transformer differential protection

Basic discussions related to the Merz-Price Scheme and its limitations which are
taken care by the biased differential scheme, are omitted for repetition

Basic considerations

a. Transformation ratio

• The nominal currents in the primary and secondary sides of the transformer
vary in inverse ratio to the corresponding voltages. This should be compensated for
by using different transformation ratios for the CTs on the primary and secondary
sides of the transformer.

b. Current Transformer Connections

• When a transformer is connected in star/delta, the secondary current has a


phase shift of 300 relative to the primary

• This phase shift can be offset by suitable secondary CT connections
• The zero-sequence currents flowing on the star-side of the transformer will
not produce current outside the delta on the other side. The zero sequence current
must therefore be eliminated from the star-side by connecting the CTs in delta.

• • The CTs on delta side should be connected in star in order to give 300 phase
shift.

• • When CTs are connected in delta, their secondary ratings must be reduced
to 1/3 times the secondary ratings of the star-connected transformer, in order that
the currents outside the delta may balance with the secondary currents of the star-
connected CTs.

• • If transformers were connected in star/star, the CTs on both sides would
need be connected in delta-delta.

c. Bias to cover tap-changing facility and CT mismatch

• If the transformer has the benefit of a tap changer, it is possible to vary its
transformation ratio for voltage control.

• • The differential protection system should be able to cope with this
variation.

• • This is because if the CTs are chosen to balance for the mean ratio of the
power transformer, a variation in ratio from the mean will create an unbalance
proportional to the ratio change. At maximum through fault current, the spill
output produced by the small percentage unbalance may be substantial

• • Differential protection should be provided with a proportional bias of an
amount which exceeds in effect the maximum ratio deviation. This stabilizes the
protection under through fault conditions while still permitting the system to have
good basic sensitivity.

d. Magnetization Inrush

• The magnetizing inrush produces a current flow into the primary winding
that does not have any equivalent in the secondary winding. The net effect is thus
similar to the situation when there is an internal fault on the transformer.

• Since the differential relay sees the magnetizing current as an internal fault,
it is necessary to have some method of distinguishing between the magnetizing
current and the fault current using one or all of the following methods.

• Using a differential relay with a suitable sensitivity to cope with the
magnetizing current, usually obtained by a unit that introduces a time delay to
cover the period of the initial inrush peak.

• • Using a harmonic-restraint unit, or a supervisory unit, in conjunction with
a differential unit.

• • Inhibiting the differential relay during the energizing the transformer.
Compared to the differential protection used in generators, there are certain
important points discussed below which must be taken care of while using such
protection for the power transformers.

1. In a power transformer, the voltage rating of the two windings is different. The
high voltage winding is low current winding while low voltage winding is high
current winding. Thus there always exists difference in current on the primary and
secondary sides of the power transformer. Hence if C.T.s of same ratio are used on
two sides, then relay may get operated through there is no fault existing.

To compensate for this difficulty, the current ratios of C.T.s on each side are
different. These ratios depend on the line currents of the power transformer and
the connection of C.T.s. Due to the different turns ratio, the currents fed into the
pilot wires from each end are same under normal conditions so that the relay
remains inoperative. For example if K is the turns ratio of a power transformer then
the ratio of C.T.s on low voltage side is made K times greater than that of C.T.s on
high voltage side.
In case of power transformers, there is an inherent phase difference between the
voltages induced in high voltage winding and low voltage winding. Due to this,
there exists a phase difference between the line currents on primary and secondary
sides of a power transformer. This introduces the phase difference between the
C.T. secondary currents, on the two sides of a power transformer. Through the
turns ratio of C.T.s are selected to compensate for turns ratio of transformer, a
differential current may result due to the phase difference between the currents
on two sides. Such a different current may operate the relay though there is no
fault. Hence it is necessary to correct the phase difference. To compensate for this,
the C.T. connections should be such that the resultant currents fed into the pilot
wires from either sides are displaced in phase by an angle equal to the phase shift
between the primary and secondary currents. To achieve this, secondaries of C.T.s
on star connected side of a power transformer are connected in delta while the
secondaries of C.T.s on delta connected side of a power transformer are connected
in star.

Buchholz relay

All faults below the oil in transformer result in the localized heating & breakdown
of the oil, some degree of arcing will always take place in a winding fault & the
resulting decomposition of it will release gases such as hydrogen, carbon monoxide
& hydrocarbons.

• When the fault is of a very minor type, such as hot joints gas is released slowly,
but a major fault involving severe arcing causes rapid release of large volumes of
gas as well as oil vapour.

• Such incipient faults of smaller or larger magnitudes can be detected by a gas


actuated relay known as Bucholtz Relay.
The Bucholtz Relay is contained in a cast housing which is connected as shown
below between the conservator tank and main tank of the transformer.
Under normal conditions, the Buchholz relay is full of oil. It consists of a cast
housing containing a hinged hollow float. A mercury switch is attached to a float.
The float being rotated in the upper part of the housing. Another hinged flap valve
is located in the lower part which is directly in the path of the oil between tank and
the conservator. Another mercury switch is attached to a flap valve. The float closes
the alarm circuit while the lower flap valve closes the trip circuit in case of internal
faults.

Operation

There are many types of internal faults such as insulation fault, core heating, bad
switch contacts, faulty joints etc. which can occur. When the fault occurs the
decomposition of oil in the main tank starts due to which the gases are generated.
As mentioned earlier, major component of such gases is hydrogen. The hydrogen
tries to rise up towards conservator but in its path it gets accumulated in the upper
part of the Buchholz relay. Through passage of the gas is prevented by the flap
valve.

When gas gets accumulated in the upper part of housing, the oil level inside the
housing falls. Due to which the hollow float tilts and closes the contacts of the
mercury switch attached to it. This completes the alarm circuit to sound an alarm.
Due to this operator knows that there is some incipient fault in the transformer.
The transformer is disconnected and the gas sample is tested. The testing results
give the indication, what type of fault is started developing in the transformer.

Hence transformer can be disconnected before grows into a serious one. The alarm
circuit does not immediately disconnect the transformer but gives only an
indication to the operator. This is because sometimes bubbles in the oil circulating
system may operate the alarm circuit even though actually there is no fault.
However if a serious fault such as internal short circuit between phases, earth fault
inside the tank etc. occurs then the considerable amount of gas gets generated. In
that case, due to a fast reduction in the level of oil, the pressure in the tank
increases. Due to this the oil rushes towards the conservator. While doing so it
passes through the relay where flap valve is present. The flap valve gets deflected
due to the rushing oil and operates the mercury switch, thereby energizing the trip
circuit which opens the circuit breaker of transformer is totally disconnected from
the supply. The connecting pipe between the tank and the conservator should be
as straight as possible and should slope upwards conservator at a small angle from
the horizontal. This angle should be around 100. For the economic considerations,
Buchholz relays are not provided for the transformer having rating below 500 KVA.

Advantages

The various advantages of the Buchholz relay are,

1. Normally a protective relay does not indicate the appearance of the fault.
It operates when fault occurs. But Buchholz relay gives an indication of the fault at
very early stage, by anticipating the fault and operating the alarm circuit. Thus the
transformer can be taken out of service before any type of serious damage occurs.

2. It is the simplest protection in case of transformers.

Limitations

The various limitation of the Buchholz relay are,

1. Can be used only for oil immersed transformers having conservator tanks.

2. Only faults below oil level are detected.

3. Setting of the mercury switches cannot be kept too sensitive otherwise the relay
can operate due to bubbles, vibration, earthquakes mechanical shocks etc.
4. The relay is slow to operate having minimum operating time of 0.1 seconds and
average time of 0.2 seconds.

Applications

The following types of transformer faults can be protected by the Buchholz relay
and are indicated by alarm:

1. Local overheating

2. Entrance of air bubbles in oil

3. Core bolt insulation failure

4. Short circuited laminations

5. Loss of oil and reduction in oil level due to leakage

6. Bad and loose electrical contacts

7. Short circuit between phases

8. Winding short circuit

9. Bushing puncture
10. Winding earth fault.

Generator protection
The range of size of generators extends from a few hundred KVA to more than 500MVA

Generator protection

INTRODUCTION

• The range of size of generators extends from a few hundred KVA to more
than 500MVA

• Small and Medium sized sets may be directly connected to the


distribution system

A larger unit is usually associated with an individual transformer, through


which the set is coupled to the EHV transmission system. No switchgear is
provided between the generator and transformer, which are treated as a unit.

Biased Differential scheme (Merz-Price Scheme) for protection of


Generators.

This is most commonly used protection scheme for the alternator stator
windings. The scheme is also called biased differential protection and
percentage differential protection. The figure below shows a schematic
arrangement of Merz-Price protection scheme for a star connected alternator.
The differential relay gives protection against short circuit fault in the stator
winding of a generator. When the neutral point of the windings is available
then, the C.T.s may be connected in star on both the phase outgoing side and
the neutral earth side, as shown in the above figure. But, if the neutral point is
not available, then the phase side CTs are connected in a residual connection,
so that it can be made suitable for comparing the current with the generator
ground point CT secondary current. The restraining coils are energized from
the secondary connection of C.T.s in each phase, through pilot wires. The
operating coils are energized by the tappings from restraining coils and the C.T.
neutral earthing connection.

The similar arrangement is used for the delta connected alternator stator
winding, as shown below.

This scheme provides very fast protection to the stator winding against phase
to phase faults and phase to ground faults. If the neutral is not grounded or
grounded through resistance then additional sensitive earth fault relay should
be provided. The advantages of this scheme are, 1. Very high speed operation
with operating time of about 15 msec. 2. It allows low fault setting which
ensures maximum protection of machine windings. 3. It ensures complete
stability under most severe through and external faults. 4. It does not require
current transformers with air gaps or special balancing features.
Earth fault protection of Generators

The neutral point of the generator is usually earthed, so as to facilitate the


protection of the stator winding and associated system. Impedance is inserted
in the earthing lead to limit the magnitude of the earth fault current. Generators
which are directly connected to the transmission or distribution system are
usually earthed through a resistance which will pass approximately rated
current to a terminal earth fault. In case of generator-transformer unit, the
generator winding and primary winding of a transformer can be treated as an
isolated system that is not influenced by the earthing requirements of the
transmission system. Modern practice is to use a large earthing transformer (5-
100 KVA) – the secondary winding which is designed for 100-500V is loaded
with a resistor of a value, which when referred through the transformer ratio,
will pass a suitable fault current. The resistor is therefore of low value and can
be of rugged construction. It is important that the earthing transformer never
becomes saturated, otherwise a very undesirable condition of ferro resonance
may occur.

Earth fault protection can be obtained by applying a relay to measure the


transformer secondary current by connecting a voltage measuring relay in
parallel with the load resistor
Generator and Transformer Unit Biased Differential Protection

In a high voltage transmission system, the bus bars are at very high voltages
than the generators. The generators are directly connected to step up
transformer to which it is connected, together from a generator transformer
unit. The protection of such a unit is achieved by differential protection scheme
using circulating current principle. While providing protection to such a unit, it
is necessary to consider the phase shift and current transformation in the step
up transformer. The figure in the following page, shows a biased differential
protection scheme used for generator transformer unit. The zone of such a
scheme includes the stator windings, the step up transformer and the
intervening connections. The transformer is delta-star hence the current
transformers on high voltage side are delta connected while those on generator
side are star connected. This cancels the displacement between line currents
introduced by the delta connected primary of the transformer. Where there is
no fault, the secondary currents of the current transformer connected on
generator side are equal to the currents in the pilot wires from the secondaries
of the delta connected current transformers on the secondary of main
transformer. When a fault occurs, the pilot wires carry the differential current
to operate the percentage differential relay.
For the protection against the earth faults, an earth fault relays is put in the
secondary winding of the main step up transformers as shown. In such a case,
differential protection acts as
a backup protection to the restricted earth fault protection. This overall
differential protection scheme does not include unit transformer as a separate
differential scheme is provided it.

PHASE FAULT

• Phase-phase faults clear of earth are less common. They may occur on
the end portion of stator coils or in the slots if the winding involves two coil
sides in the same slot. In the later case the fault will involve earth in a very short
time.

• Phase fault current is not controlled by the method of earthing the


neutral point.
INTERTURN FAULTS

• Interturn faults are also uncommon, but not unknown

• A greatest danger arising from failure to deal with interturn faults


quickly is fire. A large portion of the insulation is inflammable

Negative sequence protection

The negative sequence component can be detected by the use of a filter


network. Many negative sequence filter circuits have been evolved. One typical
negative sequence filter circuit is as follows
Basically it consists of a resistance bridge network as depicted in the first figure
showing the circuit connection. The magnitudes of the impedances of all the
branches of the network are equal. The impedances Z1 and Z3 are purely
resistive while the impedances Z2 and Z4 are the combinations of resistance
and reactance. The currents in the branches Z2 and Z4 lag by 60o from the
currents in the branches Z1 and Z3. The vertical branch B-D basically consists
of an over current element with inverse time characteristics having negligible
impedance compared to the bridge impedances.

Protection of bus bars


The protection scheme for a power system should cover the whole system against all probable types of
faults.

Protection of bus bars

The protection scheme for a power system should cover the whole system
against all probable types of faults. Unrestricted forms of line protection such
as over current and distance systems, meet this requirement, although faults in
the Bus bar zone are cleared only after some time delay. If unit protection is
applied to feeder and plant the bus bars are not inherently protected. Bus bars
have been left without specific protection. Different bus bar faults are as
follows. BUSBAR FAULTS

• Majority of bus faults involve one phase and earth, but faults arise from
many causes and a significant number are inter-phase clear of earth.

• With fully phase-segregated metal clad gear, only earth faults are
possible ,and a protective scheme need have earth fault sensitivity only.

• For outdoor busbars , protection schemes ability to respond to inter-


phase faults clear of earth is an advantage
TYPES OF PROTECTION SCHEMES

• System protection used to cover bus bars

• Frame –earth protection

• Differential protection

SYSTEM PROTECTION

• A system protection that includes over current or distance systems will


inherently give protection cover to the bus bars.

• Over current protection will only be applied to relatively simple


distribution systems, or as a back-up protection set to give considerable time
delay. Distance protection will provide cover with its second zone.

• In both cases, therefore ,the bus bar protection so obtained is slow

Frame-earth Protection

• This is purely an earth fault system, and in principle involves simply


measuring the fault current flowing from the switchgear frame to earth. To this
end a current transformer is mounted on the earthing conductor and is used to
energize a simple instantaneous relay.
This protection is nothing but the method of providing earth fault protection to
the bus bar assembly housed in a frame. This protection can be provided to the
metal clad switchgear. The arrangement is shown in the figure below. The metal
clad switchgear is lightly insulated from the earth. The enclosure of the frame
housing different switchgears and bus bars is grounded through a primary of
current transformer in between. The concrete foundation of switchgear and the
other equipments are lightly insulated from the ground. The resistance of these
equipments with earth is about 12 ohms. When there is an earth fault, then fault
current leaks from the frame and passes through the earth connection
provided. Thus the primary of C.T. senses the current due to which current
passes through the sensitive earth fault relay, thereby operating the relay.
Zones and types of Protection system
An electric power system is divided into several zones of protection. Each zone of protection, contains
one or more components of a power system in addition to two circuit breakers.

Zones and types of Protection system

Zones of Protection system

• An electric power system is divided into several zones of protection. Each


zone of protection, contains one or more components of a power system in
addition to two circuit breakers.

• When a fault occurs within the boundary of a particular zone, then the
protection system responsible for the protection of the zone acts to isolate (by
tripping the Circuit Breakers) every equipment within that zone from the rest
of the system.

• The circuit Breakers are inserted between the component of the zone and
the rest of the power system. Thus, the location of the circuit breaker helps to
define the boundaries of the zones of protection.
• Different neighbouring zones of protection are made to overlap each
other, which ensure that no part of the power system remains without
protection. However, occurrence of the fault with in the overlapped region will
initiate a tripping sequence of different circuit breakers so that the minimum
necessary to disconnect the faulty element

Types of Protection (Primary and Back-up Protection)

Primary Protection

The primary protection scheme ensures fast and selective clearing of any fault
within the boundaries of the circuit element, that the zone is required to
protect. Primary Protection as a rule is provided for each section of an electrical
installation.

However, the primary protection may fail. The primary cause of failure of the
Primary Protection system are enumerated below.
1. Current or voltage supply to the relay.

2. D.C. tripping voltage supply

3. Protective relays

4. Tripping circuit

5. Circuit Breaker

Back-up Protection

Back-up protection is the name given to a protection which backs the primary
protection whenever the later fails in operation. The back-up protection by
definition is slower than the primary protection system. The design of the back-
up protection needs to be coordinated with the design of the primary
protection and essentially it is the second line of defence after the primary
protection system.

Current Transformers(CT)s and Potential


Transformer(PT)s and their applications in protection
schemes
Current transformers are generally used to measure currents of high magnitude. These transformers
step down the current to be measured, so that it can be measured with a normal range ammeter.

CTs and PTs and their applications in protection schemes

Current transformers are generally used to measure currents of high


magnitude. These transformers step down the current to be measured, so that
it can be measured with a normal range ammeter. A Current transformer has
only one or very few number of primary turns. The primary winding may be
just a conductor or a bus bar placed in a hollow core (as shown in the figure).
The secondary winding has large number turns accurately wound for a specific
turns ratio.

Thus the current transformer steps up (increases) the voltage while stepping
down (lowering) the current. Now, the secondary current is measured with the
help of an AC ammeter. The turns ratio of a transformer is NP / NS = IS / IP

• UPS systems

• Transfer switches

• Motor-generator sets

• Commercial sub-metering,

• CT 's in one package for 3-phase metering

• Accurate measuring for metering/WATT/VAR


• Current sensing, recording, monitoring & control

• Control panels and drives

• Standard CT used as measuring standard for comparison

• Winding temperature indicator (WTI) for power transformers

• Summation current transformers.

Potential Transformer (PT)

Potential transformers are also known as voltage transformers and they are
basically step down transformers with extremely accurate turns ratio. Potential
transformers step down the voltage of high magnitude to a lower voltage which
can be measured with standard measuring instrument. These transformers
have large number of primary turns and smaller number of secondary turns. A
potential transformer is typically expressed in primary to secondary voltage
ratio. For example, a 600:120 PT would mean the voltage across secondary is
120 volts when primary voltage is 600 volts.
• Prev Page

Formation of arc during circuit breaking


The phenomena of Arc, Arc in circuit breaker, Role of arc in circuit breaker, Arc Interruption or Arc
Quenching or Arc Extinction Theory,..

Formation of arc during circuit breaking

The phenomena of Arc

During opening of current carrying contacts in a circuit breaker the medium in


between opening contacts become highly ionized through which the
interrupting current gets low resistive path and continues to flow through this
path even after the contacts are physically separated. During the flowing of
current from one contact to other the path becomes so heated that it glows in
the form of an arc.

Arc in circuit breaker

Whenever, the contacts of circuit breaker open while carrying load there is an
arc in the medium between the separating contacts of the circuit breaker. As
long as this arc is sustained in between the contacts, the current through the
circuit breaker will not be interrupted totally. For total interruption of current,
the arc needs to be quenched as quickly as possible. The main designing criteria
of a circuit breaker is to provide appropriate technology of arc quenching in
circuit breaker to fulfill quick and safe current interruption. So before going
through different arc quenching techniques employed in circuit breaker, it is
first necessary to understand the phenomena of arc in circuit breaker.

Role of arc in circuit breaker


When two current carrying contacts open, an arc bridges the contact gap
through which the current gets a low resistive path to flow so there will not be
any sudden interruption of current. As there is no sudden and abrupt change in
current during opening of the contacts, there will not be any abnormal
switching over voltage in the system. Let i is the current flowing through the
contacts just before they open and L is the system inductance, switching over
voltage during opening of contacts, may be expressed as V = L.(di/dt) where
di/dt rate of change of current with respect to time during opening of the
contacts. In the case of alternating current arc is momentarily extinguished at
every current zero. After crossing every current zero the medium between
separated contacts gets ionized again during next cycle of current and the arc
in circuit breaker is reestablished. To make the interruption complete and
successful, this re-ionization in between separated contacts to be prevented
after a current zero.

If arc in circuit breaker is absence during opening of current carrying contacts,


there would be sudden and abrupt interruption of current which will cause a
huge switching overvoltage sufficient to severely stress the insulation of the
system. On the other hand, the arc provides a gradual but quick, transition from
the current carrying to the current breaking states of the contacts.

Arc Interruption or Arc Quenching or Arc Extinction Theory

At high temperature the charged particles in a gas move rapidly and randomly,
but in absence of electric field, no net motion occurs. Whenever an electric field
is applied in the gas, the charged particles gain drift velocity superimposed on
their random thermal motion. The drift velocity is proportional to the voltage
gradient of the field and particle mobility. The particle mobility depends upon
the mass of the particle, heavier particles, lower the mobility. The mobility also
depends upon mean free paths available in the gas for random movement of the
particles. Since every time a particle collides, it loses its directed velocity and
has to be re-accelerated in the direction of electric field again. Hence net
mobility of the particles is reduced. If the medium has high pressure, it becomes
denser and hence, the gas molecules come closer to each other, therefore
collision occurs more frequently which lowers the mobility particles. The total
current by charged particles is directly proportional to their mobility.
Therefore the mobility of charged particles depends upon the temperature,
pressure of the gas and as well as nature of the gas. Again the mobility of gas
particles determines the degree ionization of gas.

So from above explanation we can say that ionization process of gas depends
upon nature of gas (heavier or lighter gas particles), pressure of gas and
temperature of gas. As we said earlier the intensity of arc column depend up on
the presence of ionized media between separated electrical contacts, hence,
special attention should be given in reducing ionization or increasing
deionization of media between contacts. That is why the main designing feature
of circuit breaker is to provide different pressure control methods, cooling
methods for different arc media in between circuit breaker contacts.

HEAT LOSS FROM ARC

Heat loss from an arc in circuit breaker takes place through conduction,
convection as well as radiation. In circuit breaker with plain break arc in oil, arc
in chutes or narrow slots nearly all the heat loss due to conduction. In air blast
circuit breaker or in breaker where a gas flow is present between the electrical
contacts, the heat loss of arc plasma occurs due to convection process. At
normal pressure the radiation is not a significant factor but at higher pressure
the radiation may become a very important factor of heat dissipation from arc
plasma. During opening of electrical contacts, the arc in circuit breaker is
produced and it is extinguished at every zero crossing, getting established again
during the next cycle. The final arc extinction or arc quenching in circuit
breaker can be achieved by rapid increase of the dielectric strength in the
medium between the contacts so that the arc gets quenched after the first zero
crossing. This rapid increase of dielectric strength in between circuit breaker
contacts is achieved either by deionization of gas in the arc media or by
replacing ionized gas by cool and fresh gas. There are various deionization
processes applied for arc extinction in circuit breaker, let us discussed in brief.
DEIONIZATION OF GAS DUE TO INCREASING PRESSURE

If pressure of the arc path increases, the density of the ionized gas is increased
which means, the particles in the gas come closer to each other and as a result
the mean free path of the particles is reduced. This increases the collision rate
and as we discussed earlier at every collision the charged particles loss their
directed velocity along electric field and again they are re-accelerated towards
field. It can be said that over all mobility of the charged particles is reduced so
the voltage required to maintain the arc is increased. Another effect of the
increased density of particles is a higher rate of deionization of gas due to the
recombination of oppositely charged particles.

The rate of ionization of gas depends upon the intensity of impact during
collision of gas particles. The intensity of impact during collision of particles
again depends upon velocity of random motions of the particles. This random
motion of a particle and its velocity increases with increase of temperature of
the gas. Hence it can be concluded like that if temperature of a gas is increased;
its ionization process is increased and opposite statement is also true that is if
the temperature is decreased the rate of ionization of gas is decreased means
deionization of gas is increased. Therefore more voltage required to maintain
arc plasma with a decreased temperature. Finally it can be said that the cooling
effectively increases the resistance of the arc.

The insulating material ( may be fluid or air) used in circuit breaker should
serve two important functions as follows:

1. It should provide sufficient insulation between the contacts when circuit


breaker opens.

2. It should extinguish the arc occurring between the contacts when circuit
breaker opens.

Methods of arc interruption


There are two methods by which interruption is done.

1. High resistance method.

2. Low resistance method or zero interruption method.

In high interruption method we can increase the electrical resistance many


times to such a high value that it forces the current to reach to zero and thus
restricting the possibility of arc to be struck again. Proper steps must be taken
in order to ensure that the rate at which the resistance is increased or
decreased is not abnormal because it may lead to generation of harmful
induced voltages in the system. The arc resistance can be increased by various
methods like lengthening or cooling of the arc etc.

Limitations of high resistance method: Arc discharge has a resistive nature due
to this most of the energy is received by circuit breaker itself hence proper care
should be taken during the manufacturing of circuit breaker like mechanical
strength etc. Therefore this method is applied in dc power circuit breaker, low
and medium ac power circuit breaker.

Low resistance method is applicable only for ac circuit and it is possible there
because of presence of natural zero of current. The arc gets extinguished at the
natural zero of the ac wave and is prevented from restricting again by rapid
building of dielectric strength of the contact space.

There are two theories which explains the phenomenon of arc extinction:

1. Energy balance theory,

2. Voltage race theory.


Before going in details about these theories, we should know the following
terms.

Restriking voltage: It may be defined as the voltage that appears across the
breaking contact at the instant of arc extinction.

Recovery voltage :

It may be defined as the voltage that appears across the breaker contact after
the complete removal of transient oscillations and final extinction of arc has
resulted in all the poles.

Active recovery voltage :

It may be defined as the instantaneous recovery voltage at the instant of arc


extinction.

Arc voltage :

It may be defined as the voltage that appears across the contact during the
arcing period, when the current flow is maintained in the form of an arc. It
assumes low value except for the point at which the voltage rise rapidly to a
peak value and current reaches to zero.

• Prev Page

AC and DC circuit breaking


DC circuit breakers and AC breaker main difference is the ability to arc. Because the exchange of each
cycle, have had zero, zero easy to extinction in the past, but has not been zero DC switching, arc
extinguishing ability is poor, so to add additional interrupter device.

AC and DC circuit breaking

DC circuit breakers and AC breaker main difference is the ability to arc. Because
the exchange of each cycle, have had zero, zero easy to extinction in the past,
but has not been zero DC switching, arc extinguishing ability is poor, so to add
additional interrupter device. DC arc is generally difficult, but the exchange has
zero, breaking easily. Exchange can be derived for the DC circuit breaker
protection, attention to three changes: 1, overload and short circuit protection.

1. long delay overload protection.

By thermal-action (double metal components) for long delay overload


protection, the source of its action as I2R, AC RMS and DC current equal to the
average, there is no need to use any restructuring. However, the large current
size, to the current transformer secondary current heat who can not be used
due to transformer can not be used on DC circuits. Release long delay if the
overload is the use of electromagnetic type (hydraulic type, that is, oil cup), then
the delayed release characteristics to change, the minimum operating current
to 110% -140% bigger, so the whole electromagnetic Release not be used for
DC circuits (such as the use will have to re-design).

2.short circuit protection.

Thermal - Magnetic AC circuit breaker short-circuit protection is the use of


magnetic system, which is used by the filtering of the rectifier circuit (DC), need
to exchange the original setting current value multiplied by a factor of
1.Electromagnetic type of short circuit protection and thermal dynamic
electromagnetic

same.

2.circuit breaker accessories, such as shunt release, under voltage release,


electrically operated institutions; shunt, under voltage are voltage coil, as long
as the line voltage, is used for systems, need not be Any change can be used for
DC system. Auxiliary and alarm contacts, AC and DC common. Electric operating
mechanism for the DC Time to re-design.

3. unlike the exchanges as DC current zero-crossing characteristics, dc short


circuit current (or even multiple small fault current) is breaking; arc out all the
difficulties, so wiring should be two extreme ways or three poles in series
increase the fracture, so that the fracture energy to bear part of the arc.

• DC arcs are to be interrupted by increasing the resistance interruption


method in which resistance of the arc is increased so that the arc voltage can no
longer maintain the current and the arc is extinguished.

• Size of DC circuit breaker increases as the voltage level increases.

• AC arcs current reduces to zero in each cycle (2 times)

• If the circuit breaker contacts are opened at time when the current passed
through zero and dielectric strength of the medium is build up rapidly so that
arc cannot strike again then arc can be extinguished successfully.

• Size of AC circuit breaker can be small compared to same voltage DC circuit


breaker.
Restriking voltage and recovery voltage
It is the transient voltage that appears across the contacts at or near current zero during arcing period.If
dielectric strength rise is greater than the rise of restriking voltage then the arc will not restrike.

Restriking voltage and recovery voltage

It is the transient voltage that appears across the contacts at or near current
zero during arcing period.If dielectric strength rise is greater than the rise of
restriking voltage then the arc will not restrike.

RestrikingVoltage :

it is the transient voltage that exists during the arcing time. ( natural frequency
kHz ).

Recovery Voltage :

it is the rms voltage after final arc extinction. ( normal frequency 50 or 60 Hz).
both voltages appear between circuit breaker poles.
• A circuit breaker is a piece of equipment which can Make or break a
circuit either manually or by remote control under normal conditions.

• Break a circuit automatically under fault condition

• Make a circuit either manually or by remote under fault condition

• Circuit Breaker consists of fixed and moving contacts called electrodes

• Under normal operating condition these contacts remain closed and will
not open automatically unless the system becomes faulty .These contacts can
be opened manually or by remote control.

• When a fault occurs in a circuit the trip coils of the circuit breaker get
energized and the moving contacts are pulled apart by some mechanism ,thus
opening the circuit.

• Prev Page

Rate of rise of recovery voltage


It is the rate of increase of restriking voltage and is abbreviated by R.R.R.V. its unit is kV/m sec.Consider
the fig2 below showing the opening of circuit breaker under fault conditions.

Rate of rise of recovery voltage

It is the rate of increase of restriking voltage and is abbreviated by R.R.R.V. its


unit is kV/m sec.Consider the fig2 below showing the opening of circuit breaker
under fault conditions. Before current interruption, the capacitance C is short
circuited by the fault and the short circuit current through the breaker is limited
by inductance L of the system
The short circuit current will lag the voltage by 90° where i represents the short
circuit current and ea represents the arc voltage. Under short circuit condition
the entire generator voltage appears across inductance L. when the contacts are
opened and the arc finally extinguishes at some current zero, the generator
voltage e is suddenly applied to the inductance and capacitance in series. This
L-C combination forms an oscillatory circuit produces a transient of
frequency;fn= 1/ [ 2π(LC)1/2] , which appears across the capacitor and hence
across the contacts of the circuit breaker. This transient voltage is known as
restriking voltage and may reach an instantaneous peak value twice the peak
phase neutral voltage i.e. 2 Em.

It is R.R.R.V, which decides whether the arc will re-strike. If R.R.R.V is greater
than the rate of rise of dielectric strength between the contacts, the arc will re-
strike. The arc will fail to re-strike if R.R.R.V is less than the rate of increase of
dielectric strength between the contacts of the breaker.

The value of R.R.R.V depends on:

• Recovery voltage

• Natural frequency of oscillations


Resistance switching
To reduce the restriking voltage, RRRV and severity of the transient oscillations, a resistance is
connected across the contacts of the circuit breaker.

Resistance switching

To reduce the restriking voltage, RRRV and severity of the transient


oscillations, a resistance is connected across the contacts of the circuit breaker.

This is known as resistance switching. The resistance is in parallel with the arc.
A part of the arc current flows through this resistance resulting in a decrease in
the arc current and increase in the deionization of the arc path and resistance
of the arc.

This process continues and the current through the shunt resistance increases
and arc current decreases. Due to the decrease in the arc current, restriking
voltage and RRRV are reduced. The resistance may be automatically switched
in with the help of a sphere gap as shown in Fig. The resistance switching is of
great help in switching out capacitive current or low inductive current.
The analysis of resistance switching can be made to find out the critical value
of the shunt resistance to obtain complete damping of transient oscillations.
Figure 5.8 shows the equivalent electrical circuit for such an analysis.

Unipolar switching

Unipolar systems usually have a dielectric that is a simple TMO. Examples are
NiO [12], CuO, CoO, Fe2O3 , HfO, TiO2Ta2O5 , Nb2O5 [10,11]. These systems
are good insulators with a large resistivity. They would normally not show any
RS effect. To get the systems into the switching regime it is usually required to
perform and initial ‗electroforming‘ step. In this process, a strong electric field
is applied, which brings the system close to the dielectric break down. A full
break down is prevented by a current limitation or compliance. After this ‗SET‘
procedure, the resistance of the device shows a significant decrease, reaching a
‗low resistance‘ state, RLO , which is stable, i.e., non-volatile. This state has an
ohmic I-V characteristic at low bias. To switch the system to the ‗high
resistance‘ state, RHI ,a voltage has to be applied to the device, with either the
same or opposite polarity than the previously applied ‗forming‘ voltage. In this
‗RESET‘ step, the resistance of the system suddenly increases, back to a ‗high
resistance‘ value close to the original one.

No current compliance should be used in the RESET step. In fact, the resistance
change occurs when the current through the device becomes larger than the
value of the compliance. To SET the system again in the low resistance state, a
voltage with current compliance has to be once again applied, similarly to the
forming step. The system‘s resistance suddenly decreases down to a value close
to RLO at a threshold voltage Vth , which is smaller that the forming one. The
SET and RESET switching process can be repeated may times. The magnitude
of resistance change typically remains within well-defined values, however
some dispersion is often observed. An example of a typical electroforming and
successive RESET and SET steps are shown in Fig

Bipolar switching

Bipolar resistive switching has been observed in a variety of ternary oxides


with perovskite structure such as SrTiO3(STO), SrZrO3 , and also in more
complex systems such as the ‗colossal‘ magnetoresistive manganites LSMO,
LCMO, PCMO, PLCMO, and even in cuprate superconductors YBCO and BSCCO.
Some reports indicate that better performance may be obtained by small
chemical substitution, such as Bi:SrTiO3 and Cr:SrTiO3 . These bipolar systems
may be either insulators or poor metals. Strong hysteresis in the two-terminal
resistance is often observed without the need of an initial forming step.
Nevertheless, electro-forming usually done, as it may improve the
reproducibility of resistive switching, but this initial forming step remains not
well understood .
The choice of a proper electrode material for each dielectric is an important
issue for bipolar devices. Sawa and collaborators have performed a systematic
study, concluding that a key feature for RS is the formation of Shottky barriers
[10]. In fact, the observed scaling of RHI and RLO with the geometry of the
devices indicate that the phenomenon should take place at the electrode/oxide
interfaces.

Consequences of occurrence of Faults

Faults are of two type · Short circuit fault- current · Open circuit fault- voltage

Consequences of occurrence of Faults

Faults are of two type

• Short circuit fault- current


• Open circuit fault- voltage

In terms of seriousness of consequences of a fault , short circuits are of far


greater concern than open circuits, although some open circuits present some
potential hazards to personnel Classification of short circuited Faults

• Three phase faults (with or without earth connection)

• Two phase faults (with or without earth connection)

• Single phase to earth faults

Classification of Open Circuit Faults

• Single Phase open Circuit

• Two phase open circuit

• Three phase open circuit


Consequences

• Damage to the equipment due to abnormally large and unbalanced currents


and low voltages produced by the short circuits

• Explosions may occur in the equipments which have insulating oil,


particularly during short circuits. This may result in fire and hazardous
conditions to personnel and equipments

• Individual generators with reduced voltage in a power station or a group of


generators operating at low voltage may lead to loss of synchronism,
subsequently resulting in islanding.

• Risk of synchronous motors in large industrial premises falling out of step and
tripping out. The general layout of a protection system may be viewed as given
in the following figure
Impedance Diagram
The impedance diagram on single-phase basis for use under balanced conditions can be easily drawn
from the SLD. The following assumptions are made in obtaining the impedance diagrams.

IMPEDANCE DIAGRAM

The impedance diagram on single-phase basis for use under balanced


conditions can be easily drawn from the SLD. The following assumptions are
made in obtaining the impedance diagrams.

Assumptions:
1. The single phase transformer equivalents are shown as ideals with
impedance on appropriate side (LV/HV),

2. The magnetizing reactance of transformers are negligible,

3. The generators are represented as constant voltage sources with series


resistance or reactance,

4. The transmission lines are approximated by their equivalent -Models,

5. The loads are assumed to be passive and are represented by a series


branch of resistance or reactance and

6. Since the balanced conditions are assumed, the neutral grounding


impedance do not appear in the impedance diagram.

Example system

As per the list of assumptions as above and with reference to the system of
figure 2, the impedance diagram can be obtained as shown in figure
Reactance Diagram
With some more additional and simplifying assumptions, the impedance diagram can be simplified
further to obtain the corresponding reactance diagram. The following are the assumptions made.

REACTANCE DIAGRAM

With some more additional and simplifying assumptions, the impedance


diagram can be simplified further to obtain the corresponding reactance
diagram. The following are the assumptions made.

Additional assumptions:

➢ The resistance is often omitted during the fault analysis. This causes a very
negligible error since, resistances are negligible

➢ Loads are Omitted

➢ Transmission line capacitances are ineffective &


➢ Magnetizing currents of transformers are neglected.

Example system

as per the assumptions given above and with reference to the system of figure
2 and Figure, the reactance diagram can be obtained as shown in figure

Per Phase and Per Unit Representation


During the power system analysis, it is a usual practice to represent current, voltage, impedance, power,
etc., of an electric power system in per unit or percentage of the base or reference value of the respective
quantities.

PER PHASE AND PER UNIT REPRESENTATION

During the power system analysis, it is a usual practice to represent current,


voltage, impedance, power, etc., of an electric power system in per unit or
percentage of the base or reference value of the respective quantities. The
numerical per unit (pu) value of any quantity is its ratio to a chosen base value
of the same dimension. Thus a pu value is a normalized quantity with respect
to the chosen base value.

Definition: Per Unit value of a given quantity is the ratio of the actual value in
any given unit to the base value in the same unit. The percent value is 100
times the pu value. Both the pu and percentage methods are simpler than the
use of actual values. Further, the main advantage in using the pu system of
computations is that the result that comes out of the sum, product, quotient,
etc. of two or more pu values is expressed in per unit itself.

Per unit value.

The per unit value of any quantity is defined as the ratio of the actual value of
the any quantity to the base value of the same quantity as a decimal.

Advantages of per unit system


i. Per unit data representation yields valuable relative magnitude
information.

ii. Circuit analysis of systems containing transformers of various


transformation ratios is greatly simplified.

iii. The p.u systems are ideal for the computerized analysis and simulation of
complex power system problems.

iv. Manufacturers usually specify the impedance values of equivalent in per


unit of the equipment rating. If the any data is not available, it is easier to
assume its per unit value than its numerical value.

v. The ohmic values of impedances are refereed to secondary is different


from the value as referee to primary. However, if base values are selected
properly, the p.u impedance is the same on the two sides of the transformer.

vi. The circuit laws are valid in p.u systems, and the power and voltages
equations are simplified since the factors of √3 and 3 are eliminated.
In an electrical power system, the parameters of interest include the current,
voltage, complex power (VA), impedance and the phase angle. Of these, the
phase angle is dimensionless and the other four quantities can be described
by knowing any two of them. Thus clearly, an arbitrary choice of any two base
values will evidently fix the other base values.

Normally the nominal voltage of lines and equipment is known along with the
complex power rating in MVA. Hence, in practice, the base values are chosen
for complex power (MVA) and line voltage (KV). The chosen base MVA is the
same for all the parts of the system. However, the base voltage is chosen with
reference to a particular section of the system and the other base voltages
(with reference to the other sections of the systems, these sections caused by
the presence of the transformers) are then related to the chosen one by the
turns-ratio of the connecting transformer.

If Ib is the base current in kilo amperes and Vb, the base voltage in kilo volts,
then the base MVA is, Sb = (VbIb). Then the base values of current & impedance
are given by

Base current (kA), Ib = MVAb/KVb


= Sb/Vb

Base impedance, Zb = (Vb/Ib) = (KVb2 / MVAb)

Hence the per unit impedance is given by Zpu = Zohms/Zb

= Zohms (MVAb/KVb2)

In 3-phase systems, KVb is the line-to-line value & MVAb is the 3-phase MVA.
[1-phase MVA = (1/3) 3-phase MVA].
• Prev Page

Change of Base
It is observed from equation that the pu value of impedance is proportional directly to the base MVA and
inversely to the square of the base KV.

CHANGE OF BASE.

It is observed from equation (3) that the pu value of impedance is


proportional directly to the base MVA and inversely to the square of the base
KV. If Zpunew is the pu impedance required to be calculated on a new set of
base values: MVAbnew & KVbnew from the already given per unit impedance
Zpuold, specified on the old set of base values, MVAbold & KVbold , then we have

Zpunew = Zpu old (MVAb new/MVAb old) (KVb old/KVb new)2

On the other hand, the change of base can also be done by first converting the
given pu impedance to its ohmic value and then calculating its pu value on the
new set of base values.

Merits and Demerits of pu System

Following are the advantages and disadvantages of adopting the pu system of


computations in electric power systems:

Merits:
➢ The pu value is the same for both 1-phase and & 3-phase systems
➢ The pu value once expressed on a proper base, will be the same when
refereed to either side of the transformer. Thus the presence of transformer
is totally eliminated

➢ The variation of values is in a smaller range 9nearby unity). Hence the


errors involved in pu computations are very less.

➢ Usually the nameplate ratings will be marked in pu on the base of the name
plate ratings, etc.

Demerits:

➢ If proper bases are not chosen, then the resulting pu values may be highly
absurd (such as 5.8 pu, -18.9 pu, etc.). This may cause confusion to the user.
However, this problem can be avoided by selecting the base MVA near the
high-rated equipment and a convenient base KV in any section of the system.

PU Impedance / Reactance Diagram

For a given power system with all its data with regard to the generators,
transformers, transmission lines, loads, etc., it is possible to obtain the
corresponding impedance or reactance diagram as explained above. If the
parametric values are shown in pu on the properly selected base values of the
system, then the diagram is referred as the per unit impedance or reactance
diagram. In forming a pu diagram, the following are the procedural steps
involved:

1. Obtain the one line diagram based on the given data


2. Choose a common base MVA for the system
3. Choose a base KV in any one section (Sections formed by transformers)
4. Find the base KV of all the sections present
5. Find pu values of all the parameters: R,X, Z, E, etc.
6. Draw the pu impedance/ reactance diagram.

Formation of Y BUS & Z BUS


The performance equations of a given power system can be considered in three different frames of
reference as discussed below:

FORMATION OF Y BUS & Z BUS

The performance equations of a given power system can be considered in three


different frames of reference as discussed below:

Frames of Reference:

Bus Frame of Reference: There are b independent equations (b = no. of buses)


relating the bus vectors of currents and voltages through the bus impedance
matrix and bus admittance matrix:

EBUS = ZBUS IBUS


IBUS = YBUS EBUS

Bus Frame of Reference: There are b independent equations (b = no. of buses)


relating the bus vectors of currents and voltages through the bus impedance
matrix and bus admittance matrix:

EBUS = ZBUS IBUS


IBUS = YBUS EBUS

Branch Frame of Reference: There are b independent equations (b = no. of


branches of a selected Tree sub-graph of the system Graph) relating the branch
vectors of currents and voltages through the branch impedance matrix and
branch admittance matrix:

EBR = ZBR IBR


IBR = YBR EBR

Loop Frame of Reference: There are b independent equations (b = no. of branches


of a selected Tree sub-graph of the system Graph) relating the branch vectors of
currents and voltages through the branch impedance matrix and branch
admittance matrix:

ELOOP = ZLOOP ILOOP


ILOOP = YLOOP ELOOP

Of the various network matrices refered above, the bus admittance matrix (YBUS )
and the bus impedance matrix (ZBUS ) are determined for a given power system
by the rule of inspection as explained next.

Rule of Inspection

Consider the 3-node admittance network as shown in figure5. Using the basic
branch relation:
I = (YV), for all the elemental currents and applying Kirchhoff’s Current Law
principle at the nodal points, we get the relations as under:

At node 1: I1 =Y1V1 + Y3 (V1-V3) + Y6 (V1 – V2)


At node 2: I2 =Y2V2 + Y5 (V2-V3) + Y6 (V2 – V1)
At node 3: 0 = Y3 (V3-V1) + Y4V3 + Y5 (V3 – V2)

These are the performance equations of the given network in admittance form
and they can be represented in matrix form as:

In other words, the relation of equation (9) can be represented in the form I BUS =
YBUS EBUS

Where, YBUS is the bus admittance matrix, I BUS & EBUS are the bus current and bus
voltage vectors respectively.

By observing the elements of the bus admittance matrix, YBUS of equation (9), it
is observed that the matrix elements can as well be obtained by a simple
inspection of the given system diagram:

Diagonal elements: A diagonal element (Yii) of the bus admittance matrix, YBUS, is
equal to the sum total of the admittance values of all the elements incident at the
bus/node i,
Off Diagonal elements: An off-diagonal element (Yij) of the bus admittance matrix,
YBUS, is equal to the negative of the admittance value of the connecting element
present between the buses I and j, if any.
This is the principle of the rule of inspection. Thus the algorithmic equations for
the rule of inspection are obtained as:
Yii = yij (j = 1,2,…….n)
Yij = - yij (j = 1,2,…….n)
For i = 1,2,….n, n = no. of buses of the given system, yij is the admittance of
element connected between buses i and j and yii is the admittance of element
connected between bus i and ground (reference bus).

Bus impedance matrix

In cases where, the bus impedance matrix is also required, then it cannot be
formed by direct inspection of the given system diagram. However, the bus
admittance matrix determined by the rule of inspection following the steps
explained above, can be inverted to obtain the bus impedance matrix, since the
two matrices are inter-invertible.

Note: It is to be noted that the rule of inspection can be applied only to those
power systems that do not have any mutually coupled elements.

EXAMPLES ON RULE OF INSPECTION:

Problem #1: Obtain the bus admittance matrix for the admittance network shown
aside by the rule of inspection
Problem #2: Obtain YBUS and ZBUS matrices for the impedance network shown
aside by the rule of inspection. Also, determine YBUS for the reduced network
after eliminating the eligible unwanted node. Draw the resulting reduced system
diagram.
EXAMPLES ON PER UNIT ANALYSIS:

Problem #1:

Two generators rated 10 MVA, 13.2 KV and 15 MVA, 13.2 KV are connected in
parallel to a bus bar. They feed supply to 2 motors of inputs 8 MVA and 12 MVA
respectively.
The operating voltage of motors is 12.5 KV. Assuming the base quantities as 50
MVA, 13.8 KV, draw the per unit reactance diagram. The percentage reactance
for generators is 15% and that for motors is 20%.

Solution:
The one line diagram with the data is obtained as shown in figure

• Prev Page

• The operating voltage of motors is 12.5 KV. Assuming the base quantities as 50
MVA, 13.8 KV, draw the per unit reactance diagram. The percentage reactance
for generators is 15% and that for motors is 20%.

• Solution:
• The one line diagram with the data is obtained as shown in figure




• Problem #1:

• Two generators rated 10 MVA, 13.2 KV and 15 MVA, 13.2 KV are connected
in parallel to a bus bar. They feed supply to 2 motors of inputs 8 MVA and 12
MVA respectively. The operating voltage of motors is 12.5 KV. Assuming the
base quantities as 50 MVA, 13.8 KV, draw the per unit reactance diagram. The
percentage reactance for generators is 15% and that for motors is 20%.

• Solution:

• The one line diagram with the data is obtained as shown in figure P1(a).

• Selection of base quantities: 50 MVA, 13.8 KV (Given)

• Calculation of pu values:
• XG1 = j 0.15 (50/10) (13.2/13.8)2 = j 0.6862 pu.
• XG2 = j 0.15 (50/15) (13.2/13.8)2 = j 0.4574 pu.
• Xm1 = j 0.2 (50/8) (12.5/13.8)2 = j 1.0256 pu.
• Xm2 = j 0.2 (50/12) (12.5/13.8)2 = j 0.6837 pu.

• Eg1 = Eg2 = (13.2/13.8) = 0.9565 ∠00 pu


• Em1 = Em2 = (12.5/13.8) = 0.9058 ∠00 pu


• Thus the pu reactance diagram can be drawn as shown in figure P1(b).


• Problem #2:

• Draw the per unit reactance diagram for the system shown in figure below.
Choose a base of 11 KV, 100 MVA in the generator circuit.


• Solution:

• The one line diagram with the data is considered as shown in figure.

• Selection of base quantities:


• 100 MVA, 11 KV in the generator circuit(Given); the voltage bases in other


sections are: 11 (115/11.5) = 110 KV in the transmission line circuit and 110
(6.6/11.5) = 6.31 KV in the motor circuit.

• Calculation of pu values:
• Calculation of pu values:
• XG = j 0.1 pu, Xm = j 0.2 (100/90) (6.6/6.31)2 = j 0.243 pu.
• Xt1 =Xt2 = j 0.1 (100/50) (11.5/11)2 = j 0.2185 pu.
• Xt3 =Xt4 = j 0.1 (100/50) (6.6/6.31)2 = j 0.219 pu.
• Xlines = j 20 (100/1102) = j 0.1652 pu.
• Eg = 1.0∠00 pu, Em = (6.6/6.31) = 1.045∠00 pu
• Thus the pu reactance diagram can be drawn as shown in figure P2(b).


• Problem #3:

• A 30 MVA, 13.8 KV, 3-phase generator has a sub transient reactance of 15%.
The generator supplies 2 motors through a step-up transformer - transmission
line – step-down transformer arrangement. The motors have rated inputs of 20
MVA and 10 MVA at 12.8 KV with 20% sub transient reactance each. The 3-
phase

• transformers are rated at 35 MVA, 13.2 KV -∆ /115 KV-Y with 10 % leakage


reactance. The line reactance is 80 ohms. Draw the equivalent per unit reactance
diagram by selecting the generator ratings as base values in the generator circuit.

• Solution:

• The one line diagram with the data is obtained as shown in figure P3(a).

• Selection of base quantities:

• 30 MVA, 13.8 KV in the generator circuit(Given); The voltage bases in other


sections are:

• 13.8(115/13.2) = 120.23 KV in the transmission line circuit and


• 120.23(13.26/115) = 13.8 KV in the motor circuit.


• Calculation of pu values:

• XG = j 0.15 pu.

• Xm1 = j 0.2 (30/20) (12.8/13.8)2 = j 0.516 pu.


• Xm2 = j 0.2 (30/10) (12.8/13.8)2 = j 0.2581 pu.


• Xt1 =Xt2 = j 0.1 (30/35) (13.2/13.8)2 = j 0.0784 pu.


• Xline = j 80 (30/120.232) = j 0.17 pu.



• Eg = 1.0∠00 pu; Em1 = Em2 = (6.6/6.31) = 0.93∠00 pu

• Thus the pu reactance diagram can be drawn as shown in figure P3(b).


• Problem #4:

• A 33 MVA, 13.8 KV, 3-phase generator has a sub transient reactance of 0.5%.
The generator supplies a motor through a step- up transformer - transmission
line – step-down transformer arrangement. The motor has rated input of 25
MVA at 6.6 KV with 25% sub transient reactance. Draw the equivalent per unit
impedance diagram by selecting 25 MVA (3φ), 6.6 KV (LL) as base values in
the motor circuit, given the transformer and transmission line data as under:

• Step up transformer bank: three single phase units, connected ∆–Y, each rated
10 MVA, 13.2/6.6 KV with 7.7 % leakage reactance and 0.5 % leakage
resistance;

• Transmission line: 75 KM long with a positive sequence reactance of 0.8 ohm/


KM and a resistance of 0.2 ohm/ KM; and

• Step down transformer bank: three single phase units, connected ∆–Y, each
rated 8.33 MVA, 110/3.98 KV with 8% leakage reactance and 0.8 % leakage
resistance;

• Solution:

• The one line diagram with the data is obtained as shown in figure P4(a).


• 3-phase ratings of transformers:

• T1: 3(10) = 30 MVA, 13.2/ 66.4√3 KV = 13.2/ 115 KV, X = 0.077, R = 0.005
pu.
• T2: 3(8.33) = 25 MVA, 110/ 3.98√3 KV = 110/ 6.8936 KV, X = 0.08, R =
0.008 pu.

• Selection of base quantities:


• 25 MVA, 6.6 KV in the motor circuit (Given); the voltage bases in other sections
are: 6.6 (110/6.8936) = 105.316 KV in the transmission line circuit and 105.316
(13.2/115) = 12.09 KV in the generator circuit.

• Calculation of pu values:
• Xm = j 0.25 pu; Em = 1.0∠00 pu.

• XG = j 0.005 (25/33) (13.8/12.09)2 = j 0.005 pu; Eg = 13.8/12.09 = 1.414∠00 pu.


• Zt1 = 0.005 + j 0.077 (25/30) (13.2/12.09)2 = 0.005 + j 0.0765 pu. (ref. to LV


side)
• Zt2 = 0.008 + j 0.08 (25/25) (110/105.316)2 = 0.0087 + j 0.0873 pu. (ref. to HV
side)
• Zline = 75 (0.2+j 0.8) (25/ 105.3162) = 0.0338 + j 0.1351 pu.



• Problems

• 1. Determine the reactances of the three generators rated as follows on a


common base of 200 MVA, 35 KV: Generator 1: 100 MVA, 33 KV, sub
transient reactance of 10%; Generator 2: 150 MVA, 32 KV, sub transient
reactance of 8% and Generator 3: 110 MVA, 30 KV, sub transient reactance of
12%.

• [Answers: XG1 = j 0.1778, Xg2 = j 0.089, Xg3 = j 0.16 all in per unit]

• 2. A 100 MVA, 33 KV, 3-phase generator has a sub transient reactance of 15%.
The generator supplies 3 motors through a step-up transformer - transmission
line – step- down transformer arrangement. The motors have rated inputs of 30
MVA, 20 MVA and 50 MVA, at 30 KV with 20% sub transient reactance each.
The 3 -phase transformers are rated at 100 MVA, 32 KV-∆ /110 KV-Y with 8
% leakage reactance. The line has a reactance of 50 ohms. By selecting the
generator ratings as base values in the generator circuit, determine the base
values in all the other parts of the system. Hence evaluate the corresponding pu
values and draw the equivalent per unit reactance diagram.

• [Answers: X G = j 0.15, Xm1 = j 0.551, Xm2 = j 0.826, Xm3 = j 0.331, Eg1=1.0 ∠00,
Em1 = Em2 = Em3 = 0.91∠00, Xt1 = Xt2 = j 0.0775 and Xline = j 0.39 all in per unit]

• 3. A 80 MVA, 10 KV, 3-phase generator has a sub transient reactance of 10%.


The generator supplies a motor through a step -up transformer - transmission
line – step-down transformer arrangement. The motor has rated input of 95
MVA, 6.3 KV with 15% sub transient reactance. The step-up 3-phase
transformer is rated at 90 MVA, 11 KV-Y /110 KV-Y with 10% leakage
reactance. The 3-phase step-down transformer consists of three single phase Y
-∆ connected transformers, each rated at 33.33 MVA, 68/6.6 KV with 10%
leakage reactance. The line has a reactance of 20 ohms. By selecting the 11 KV,
100 MVA as base values in the generator circuit, determine the base values in
all the other parts of the system. Hence evaluate the corresponding pu values
and draw the equivalent per unit reactance diagram.

• [Answers: XG = j 1.103, Xm = j 0.165, Eg1=0.91∠00, Em= 1.022∠00, Xt1 = j 0.11,


Xt2 = j 0.114 and Xline = j 0.17 all in per unit]

• 4. For the three-phase system shown below, draw an impedance diagram


expressing all impedances in per unit on a common base of 20 MVA, 2600 V
on the HV side of the transformer. Using this impedance diagram, find the HV
and LV currents.


• [Answers: Sb = 20 MVA; Vb=2.6 KV (HV) and 0.2427 KV (LV); Vt=1.0∠00,


Xt = j 0.107, Zcable = 0.136 +j 0.204 and Zload = 5.66 + j 2.26, I = 0.158 all in per
unit, I (hv)= 0.7 A and I (lv) = 7.5 A]
• The operating voltage of motors is 12.5 KV. Assuming the base quantities as 50
MVA, 13.8 KV, draw the per unit reactance diagram. The percentage reactance
for generators is 15% and that for motors is 20%.

• Solution:
• The one line diagram with the data is obtained as shown in figure




• Problem #1:

• Two generators rated 10 MVA, 13.2 KV and 15 MVA, 13.2 KV are connected
in parallel to a bus bar. They feed supply to 2 motors of inputs 8 MVA and 12
MVA respectively. The operating voltage of motors is 12.5 KV. Assuming the
base quantities as 50 MVA, 13.8 KV, draw the per unit reactance diagram. The
percentage reactance for generators is 15% and that for motors is 20%.

• Solution:

• The one line diagram with the data is obtained as shown in figure P1(a).


• Selection of base quantities: 50 MVA, 13.8 KV (Given)

• Calculation of pu values:
• XG1 = j 0.15 (50/10) (13.2/13.8)2 = j 0.6862 pu.
• XG2 = j 0.15 (50/15) (13.2/13.8)2 = j 0.4574 pu.
• Xm1 = j 0.2 (50/8) (12.5/13.8)2 = j 1.0256 pu.
• Xm2 = j 0.2 (50/12) (12.5/13.8)2 = j 0.6837 pu.

• Eg1 = Eg2 = (13.2/13.8) = 0.9565 ∠00 pu


• Em1 = Em2 = (12.5/13.8) = 0.9058 ∠00 pu


• Thus the pu reactance diagram can be drawn as shown in figure P1(b).



• Problem #2:

• Draw the per unit reactance diagram for the system shown in figure below.
Choose a base of 11 KV, 100 MVA in the generator circuit.


• Solution:

• The one line diagram with the data is considered as shown in figure.

• Selection of base quantities:


• 100 MVA, 11 KV in the generator circuit(Given); the voltage bases in other


sections are: 11 (115/11.5) = 110 KV in the transmission line circuit and 110
(6.6/11.5) = 6.31 KV in the motor circuit.

• Calculation of pu values:
• Calculation of pu values:
• XG = j 0.1 pu, Xm = j 0.2 (100/90) (6.6/6.31)2 = j 0.243 pu.
• Xt1 =Xt2 = j 0.1 (100/50) (11.5/11)2 = j 0.2185 pu.
• Xt3 =Xt4 = j 0.1 (100/50) (6.6/6.31)2 = j 0.219 pu.
• Xlines = j 20 (100/1102) = j 0.1652 pu.
• Eg = 1.0∠00 pu, Em = (6.6/6.31) = 1.045∠00 pu
• Thus the pu reactance diagram can be drawn as shown in figure P2(b).


• Problem #3:

• A 30 MVA, 13.8 KV, 3-phase generator has a sub transient reactance of 15%.
The generator supplies 2 motors through a step-up transformer - transmission
line – step-down transformer arrangement. The motors have rated inputs of 20
MVA and 10 MVA at 12.8 KV with 20% sub transient reactance each. The 3-
phase

• transformers are rated at 35 MVA, 13.2 KV -∆ /115 KV-Y with 10 % leakage


reactance. The line reactance is 80 ohms. Draw the equivalent per unit reactance
diagram by selecting the generator ratings as base values in the generator circuit.

• Solution:

• The one line diagram with the data is obtained as shown in figure P3(a).

• Selection of base quantities:

• 30 MVA, 13.8 KV in the generator circuit(Given); The voltage bases in other


sections are:

• 13.8(115/13.2) = 120.23 KV in the transmission line circuit and


• 120.23(13.26/115) = 13.8 KV in the motor circuit.


• Calculation of pu values:

• XG = j 0.15 pu.

• Xm1 = j 0.2 (30/20) (12.8/13.8)2 = j 0.516 pu.


• Xm2 = j 0.2 (30/10) (12.8/13.8)2 = j 0.2581 pu.


• Xt1 =Xt2 = j 0.1 (30/35) (13.2/13.8)2 = j 0.0784 pu.


• Xline = j 80 (30/120.232) = j 0.17 pu.



• Eg = 1.0∠00 pu; Em1 = Em2 = (6.6/6.31) = 0.93∠00 pu

• Thus the pu reactance diagram can be drawn as shown in figure P3(b).


• Problem #4:

• A 33 MVA, 13.8 KV, 3-phase generator has a sub transient reactance of 0.5%.
The generator supplies a motor through a step- up transformer - transmission
line – step-down transformer arrangement. The motor has rated input of 25
MVA at 6.6 KV with 25% sub transient reactance. Draw the equivalent per unit
impedance diagram by selecting 25 MVA (3φ), 6.6 KV (LL) as base values in
the motor circuit, given the transformer and transmission line data as under:

• Step up transformer bank: three single phase units, connected ∆–Y, each rated
10 MVA, 13.2/6.6 KV with 7.7 % leakage reactance and 0.5 % leakage
resistance;

• Transmission line: 75 KM long with a positive sequence reactance of 0.8 ohm/


KM and a resistance of 0.2 ohm/ KM; and

• Step down transformer bank: three single phase units, connected ∆–Y, each
rated 8.33 MVA, 110/3.98 KV with 8% leakage reactance and 0.8 % leakage
resistance;

• Solution:

• The one line diagram with the data is obtained as shown in figure P4(a).


• 3-phase ratings of transformers:

• T1: 3(10) = 30 MVA, 13.2/ 66.4√3 KV = 13.2/ 115 KV, X = 0.077, R = 0.005
pu.
• T2: 3(8.33) = 25 MVA, 110/ 3.98√3 KV = 110/ 6.8936 KV, X = 0.08, R =
0.008 pu.

• Selection of base quantities:


• 25 MVA, 6.6 KV in the motor circuit (Given); the voltage bases in other sections
are: 6.6 (110/6.8936) = 105.316 KV in the transmission line circuit and 105.316
(13.2/115) = 12.09 KV in the generator circuit.

• Calculation of pu values:
• Xm = j 0.25 pu; Em = 1.0∠00 pu.

• XG = j 0.005 (25/33) (13.8/12.09)2 = j 0.005 pu; Eg = 13.8/12.09 = 1.414∠00 pu.


• Zt1 = 0.005 + j 0.077 (25/30) (13.2/12.09)2 = 0.005 + j 0.0765 pu. (ref. to LV


side)
• Zt2 = 0.008 + j 0.08 (25/25) (110/105.316)2 = 0.0087 + j 0.0873 pu. (ref. to HV
side)
• Zline = 75 (0.2+j 0.8) (25/ 105.3162) = 0.0338 + j 0.1351 pu.



• Problems

• 1. Determine the reactances of the three generators rated as follows on a


common base of 200 MVA, 35 KV: Generator 1: 100 MVA, 33 KV, sub
transient reactance of 10%; Generator 2: 150 MVA, 32 KV, sub transient
reactance of 8% and Generator 3: 110 MVA, 30 KV, sub transient reactance of
12%.

• [Answers: XG1 = j 0.1778, Xg2 = j 0.089, Xg3 = j 0.16 all in per unit]

• 2. A 100 MVA, 33 KV, 3-phase generator has a sub transient reactance of 15%.
The generator supplies 3 motors through a step-up transformer - transmission
line – step- down transformer arrangement. The motors have rated inputs of 30
MVA, 20 MVA and 50 MVA, at 30 KV with 20% sub transient reactance each.
The 3 -phase transformers are rated at 100 MVA, 32 KV-∆ /110 KV-Y with 8
% leakage reactance. The line has a reactance of 50 ohms. By selecting the
generator ratings as base values in the generator circuit, determine the base
values in all the other parts of the system. Hence evaluate the corresponding pu
values and draw the equivalent per unit reactance diagram.

• [Answers: X G = j 0.15, Xm1 = j 0.551, Xm2 = j 0.826, Xm3 = j 0.331, Eg1=1.0 ∠00,
Em1 = Em2 = Em3 = 0.91∠00, Xt1 = Xt2 = j 0.0775 and Xline = j 0.39 all in per unit]

• 3. A 80 MVA, 10 KV, 3-phase generator has a sub transient reactance of 10%.


The generator supplies a motor through a step -up transformer - transmission
line – step-down transformer arrangement. The motor has rated input of 95
MVA, 6.3 KV with 15% sub transient reactance. The step-up 3-phase
transformer is rated at 90 MVA, 11 KV-Y /110 KV-Y with 10% leakage
reactance. The 3-phase step-down transformer consists of three single phase Y
-∆ connected transformers, each rated at 33.33 MVA, 68/6.6 KV with 10%
leakage reactance. The line has a reactance of 20 ohms. By selecting the 11 KV,
100 MVA as base values in the generator circuit, determine the base values in
all the other parts of the system. Hence evaluate the corresponding pu values
and draw the equivalent per unit reactance diagram.

• [Answers: XG = j 1.103, Xm = j 0.165, Eg1=0.91∠00, Em= 1.022∠00, Xt1 = j 0.11,


Xt2 = j 0.114 and Xline = j 0.17 all in per unit]

• 4. For the three-phase system shown below, draw an impedance diagram


expressing all impedances in per unit on a common base of 20 MVA, 2600 V
on the HV side of the transformer. Using this impedance diagram, find the HV
and LV currents.


• [Answers: Sb = 20 MVA; Vb=2.6 KV (HV) and 0.2427 KV (LV); Vt=1.0∠00,


Xt = j 0.107, Zcable = 0.136 +j 0.204 and Zload = 5.66 + j 2.26, I = 0.158 all in per
unit, I (hv)= 0.7 A and I (lv) = 7.5 A]
• The operating voltage of motors is 12.5 KV. Assuming the base quantities as 50
MVA, 13.8 KV, draw the per unit reactance diagram. The percentage reactance
for generators is 15% and that for motors is 20%.

• Solution:
• The one line diagram with the data is obtained as shown in figure




• Problem #1:

• Two generators rated 10 MVA, 13.2 KV and 15 MVA, 13.2 KV are connected
in parallel to a bus bar. They feed supply to 2 motors of inputs 8 MVA and 12
MVA respectively. The operating voltage of motors is 12.5 KV. Assuming the
base quantities as 50 MVA, 13.8 KV, draw the per unit reactance diagram. The
percentage reactance for generators is 15% and that for motors is 20%.

• Solution:

• The one line diagram with the data is obtained as shown in figure P1(a).


• Selection of base quantities: 50 MVA, 13.8 KV (Given)

• Calculation of pu values:
• XG1 = j 0.15 (50/10) (13.2/13.8)2 = j 0.6862 pu.
• XG2 = j 0.15 (50/15) (13.2/13.8)2 = j 0.4574 pu.
• Xm1 = j 0.2 (50/8) (12.5/13.8)2 = j 1.0256 pu.
• Xm2 = j 0.2 (50/12) (12.5/13.8)2 = j 0.6837 pu.

• Eg1 = Eg2 = (13.2/13.8) = 0.9565 ∠00 pu


• Em1 = Em2 = (12.5/13.8) = 0.9058 ∠00 pu


• Thus the pu reactance diagram can be drawn as shown in figure P1(b).



• Problem #2:

• Draw the per unit reactance diagram for the system shown in figure below.
Choose a base of 11 KV, 100 MVA in the generator circuit.


• Solution:

• The one line diagram with the data is considered as shown in figure.

• Selection of base quantities:


• 100 MVA, 11 KV in the generator circuit(Given); the voltage bases in other


sections are: 11 (115/11.5) = 110 KV in the transmission line circuit and 110
(6.6/11.5) = 6.31 KV in the motor circuit.

• Calculation of pu values:
• Calculation of pu values:
• XG = j 0.1 pu, Xm = j 0.2 (100/90) (6.6/6.31)2 = j 0.243 pu.
• Xt1 =Xt2 = j 0.1 (100/50) (11.5/11)2 = j 0.2185 pu.
• Xt3 =Xt4 = j 0.1 (100/50) (6.6/6.31)2 = j 0.219 pu.
• Xlines = j 20 (100/1102) = j 0.1652 pu.
• Eg = 1.0∠00 pu, Em = (6.6/6.31) = 1.045∠00 pu
• Thus the pu reactance diagram can be drawn as shown in figure P2(b).


• Problem #3:

• A 30 MVA, 13.8 KV, 3-phase generator has a sub transient reactance of 15%.
The generator supplies 2 motors through a step-up transformer - transmission
line – step-down transformer arrangement. The motors have rated inputs of 20
MVA and 10 MVA at 12.8 KV with 20% sub transient reactance each. The 3-
phase

• transformers are rated at 35 MVA, 13.2 KV -∆ /115 KV-Y with 10 % leakage


reactance. The line reactance is 80 ohms. Draw the equivalent per unit reactance
diagram by selecting the generator ratings as base values in the generator circuit.

• Solution:

• The one line diagram with the data is obtained as shown in figure P3(a).

• Selection of base quantities:

• 30 MVA, 13.8 KV in the generator circuit(Given); The voltage bases in other


sections are:

• 13.8(115/13.2) = 120.23 KV in the transmission line circuit and


• 120.23(13.26/115) = 13.8 KV in the motor circuit.


• Calculation of pu values:

• XG = j 0.15 pu.

• Xm1 = j 0.2 (30/20) (12.8/13.8)2 = j 0.516 pu.


• Xm2 = j 0.2 (30/10) (12.8/13.8)2 = j 0.2581 pu.


• Xt1 =Xt2 = j 0.1 (30/35) (13.2/13.8)2 = j 0.0784 pu.


• Xline = j 80 (30/120.232) = j 0.17 pu.



• Eg = 1.0∠00 pu; Em1 = Em2 = (6.6/6.31) = 0.93∠00 pu

• Thus the pu reactance diagram can be drawn as shown in figure P3(b).


• Problem #4:

• A 33 MVA, 13.8 KV, 3-phase generator has a sub transient reactance of 0.5%.
The generator supplies a motor through a step- up transformer - transmission
line – step-down transformer arrangement. The motor has rated input of 25
MVA at 6.6 KV with 25% sub transient reactance. Draw the equivalent per unit
impedance diagram by selecting 25 MVA (3φ), 6.6 KV (LL) as base values in
the motor circuit, given the transformer and transmission line data as under:

• Step up transformer bank: three single phase units, connected ∆–Y, each rated
10 MVA, 13.2/6.6 KV with 7.7 % leakage reactance and 0.5 % leakage
resistance;

• Transmission line: 75 KM long with a positive sequence reactance of 0.8 ohm/


KM and a resistance of 0.2 ohm/ KM; and

• Step down transformer bank: three single phase units, connected ∆–Y, each
rated 8.33 MVA, 110/3.98 KV with 8% leakage reactance and 0.8 % leakage
resistance;

• Solution:

• The one line diagram with the data is obtained as shown in figure P4(a).


• 3-phase ratings of transformers:

• T1: 3(10) = 30 MVA, 13.2/ 66.4√3 KV = 13.2/ 115 KV, X = 0.077, R = 0.005
pu.
• T2: 3(8.33) = 25 MVA, 110/ 3.98√3 KV = 110/ 6.8936 KV, X = 0.08, R =
0.008 pu.

• Selection of base quantities:


• 25 MVA, 6.6 KV in the motor circuit (Given); the voltage bases in other sections
are: 6.6 (110/6.8936) = 105.316 KV in the transmission line circuit and 105.316
(13.2/115) = 12.09 KV in the generator circuit.

• Calculation of pu values:
• Xm = j 0.25 pu; Em = 1.0∠00 pu.

• XG = j 0.005 (25/33) (13.8/12.09)2 = j 0.005 pu; Eg = 13.8/12.09 = 1.414∠00 pu.


• Zt1 = 0.005 + j 0.077 (25/30) (13.2/12.09)2 = 0.005 + j 0.0765 pu. (ref. to LV


side)
• Zt2 = 0.008 + j 0.08 (25/25) (110/105.316)2 = 0.0087 + j 0.0873 pu. (ref. to HV
side)
• Zline = 75 (0.2+j 0.8) (25/ 105.3162) = 0.0338 + j 0.1351 pu.



• Problems

• 1. Determine the reactances of the three generators rated as follows on a


common base of 200 MVA, 35 KV: Generator 1: 100 MVA, 33 KV, sub
transient reactance of 10%; Generator 2: 150 MVA, 32 KV, sub transient
reactance of 8% and Generator 3: 110 MVA, 30 KV, sub transient reactance of
12%.

• [Answers: XG1 = j 0.1778, Xg2 = j 0.089, Xg3 = j 0.16 all in per unit]

• 2. A 100 MVA, 33 KV, 3-phase generator has a sub transient reactance of 15%.
The generator supplies 3 motors through a step-up transformer - transmission
line – step- down transformer arrangement. The motors have rated inputs of 30
MVA, 20 MVA and 50 MVA, at 30 KV with 20% sub transient reactance each.
The 3 -phase transformers are rated at 100 MVA, 32 KV-∆ /110 KV-Y with 8
% leakage reactance. The line has a reactance of 50 ohms. By selecting the
generator ratings as base values in the generator circuit, determine the base
values in all the other parts of the system. Hence evaluate the corresponding pu
values and draw the equivalent per unit reactance diagram.

• [Answers: X G = j 0.15, Xm1 = j 0.551, Xm2 = j 0.826, Xm3 = j 0.331, Eg1=1.0 ∠00,
Em1 = Em2 = Em3 = 0.91∠00, Xt1 = Xt2 = j 0.0775 and Xline = j 0.39 all in per unit]

• 3. A 80 MVA, 10 KV, 3-phase generator has a sub transient reactance of 10%.


The generator supplies a motor through a step -up transformer - transmission
line – step-down transformer arrangement. The motor has rated input of 95
MVA, 6.3 KV with 15% sub transient reactance. The step-up 3-phase
transformer is rated at 90 MVA, 11 KV-Y /110 KV-Y with 10% leakage
reactance. The 3-phase step-down transformer consists of three single phase Y
-∆ connected transformers, each rated at 33.33 MVA, 68/6.6 KV with 10%
leakage reactance. The line has a reactance of 20 ohms. By selecting the 11 KV,
100 MVA as base values in the generator circuit, determine the base values in
all the other parts of the system. Hence evaluate the corresponding pu values
and draw the equivalent per unit reactance diagram.

• [Answers: XG = j 1.103, Xm = j 0.165, Eg1=0.91∠00, Em= 1.022∠00, Xt1 = j 0.11,


Xt2 = j 0.114 and Xline = j 0.17 all in per unit]

• 4. For the three-phase system shown below, draw an impedance diagram


expressing all impedances in per unit on a common base of 20 MVA, 2600 V
on the HV side of the transformer. Using this impedance diagram, find the HV
and LV currents.


• [Answers: Sb = 20 MVA; Vb=2.6 KV (HV) and 0.2427 KV (LV); Vt=1.0∠00,


Xt = j 0.107, Zcable = 0.136 +j 0.204 and Zload = 5.66 + j 2.26, I = 0.158 all in per
unit, I (hv)= 0.7 A and I (lv) = 7.5 A]
Important Short Questions and Answers:
Power System Analysis - Introduction
Power System Analysis - Introduction - Important Short Questions and Answers: Power System
Analysis - Introduction

1. What is single line diagram?

A single line diagram is a diagrammatic representation of power


system in which the components are represented by their symbols
and the interconnection between them are shown by a single straight
line.

2. What are the components of a power system?

The components of a power system are Generators, Power


transformers, Transmission lines, Substation transformers,
Distribution transformers and Loads.

3. Define per unit value.


The per unit value of any quantity is defined as the ratio of
actual value of the quantity to base value of the quantity.

Per unit = Actual value / Base value

4. What is the need for base value?

The components or various sections of power system


may operate at different voltage and power levels. It will be
convenient for analysis of power system if the voltage, power, current
and impedance ratings of components of power system are expressed
with a common value called base value. Hence for analysis purpose a
base value is chosen for voltage, power, current and impedance.

5. Write the equation for converting the p.u. impedance expressed in one base to
another.

Zp.u,new = Zp.u,old) * (kVb,old / kVb,old) * (MVAb,new / MVAb,old)

6. What are the advantages of per unit computations?


• manufacturers usually specify the impedance of a device or machine in per
unit on the basis of the name plate details.

• The p.u. values of widely different rating machines lie within a narrow
range eventhough the ohmic values has a very large range.

• The p.u. impedance of circuit element connected by a transformer


expressed on a proper base will be same if it is referred to either side of a
transformer.

• The p.u. impedance of a 3-phase transformer is independent of the type


winding connection.

7. What are the approximations made in impedance diagram?

• The neutral reactances are neglected.


• The shunt branches in equivalent circuit of induction motor are neglected.

8. What are the approximations made in reactance diagram?

▪ The neutral reactance are neglected.


o The shunt branches in equivalent circuit of induction motor are
neglected.
o The resistances are neglected.
o All static loads and induction motor are neglected.
o The capacitances of the transmission lines are neglected.

9. Give the equations for transforming base kV on LV side to HV side


and viceversa. Base kV on HT side = Base kV on LT side * (HT voltage
rating / LT voltage rating) Base kV on LT side = Base kV on HT side *
(LT voltage rating / HT voltage rating)

MODELLING OF VARIOUS COMPONENTS/ACCESSORIES

1. What is a bus?

The meeting point of various components in a power system is called


a bus.
2. What is bus admittance matrix?

The matrix consisting of the self and mutual admittance of the


network of a power system is called bus admittance matrix.

3. Name the diagonal and off-diagonal elements of bus admittance matrix.

The diagonal elements of bus admittance matrix are called self


admittances of the matrix and off-diagonal elements are called mutual
admittances of the buses.

4. Write the equation to find the elements of new bus admittance


matrix after eliminating nth row and column in a n*n matrix.

Yjk = Yjk-(YjnYnk / Ynn)

5. What is bus impedance matrix?

The matrix consisting of driving point impedances and transfer


impedances of the network of a power system is called bus impedance
matrix.
6. Name the diagonal and off-diagonal elements of bus impedance matrix.

The diagonal elements of bus impedance matrix are called driving


point impedances of the buses and off-diagonal elements are called
transfer impedances of the buses.

7. What are the methods available for forming bus impedances matrix?

(1)Form the bus admittances matrix and then take its inverse to get bus
impedance matrix.

(2)Directly form the bus impedance matrix from the reactance


diagram. This method utilizes the techniques of modification of existing
bus impedance matrix due to addition of new bus.

8. Write the four ways of adding an impedance to an existing system so


as to modify bus impedance matrix.
Case 1:Adding a branch impedance Zb from a new bus P to the
reference bus. Case 2:Adding a branch impedance Z b from a new
bus P to the existing bus q. Case 3:Adding a branch impedance Z b
from a existing bus q to the reference bus.

Case 4:Adding a branch impedance Zb between two existing bus h and q.

9. What is off-nominal transformer ratio?

When the voltage or turns ratio of a transformer is not used to


decide the ratio of base kV then its voltage or turns ratio is called off-
nominal turns ratio.

Importance of Power Flow Analysis in Planning and


Operation of Power Systems
POWER FLOW STUDY OR LOAD FLOW STUDY, Information’s that are obtained from a load flow study,
Need for load flow study

IMPORTANCE OF POWER FLOW ANALYSIS IN PLANNING


AND OPERATION OF POWER SYSTEMS

POWER FLOW STUDY OR LOAD FLOW STUDY


The study of various methods of solution to power system network is referred
to as load flow study. The solution provides the voltages at various buses,
power flowing in various lines and line losses.

Information’s that are obtained from a load flow study

The information obtained from a load flow study is magnitude and phase
angle of voltages, real and reactive power flowing in each line and the line
losses. The load flow solution also gives the initial conditions of the system
when the transient behavior of the system is to be studied.

Need for load flow study

The load flow study of a power system is essential to decide the best operation
of existing system and for planning the future expansion of the system. It is
also essential for designing a new power system.

Statement of Power Flow Problem


Quantities associated with each bus in a system, Work involved (or) to be performed by a load flow
study

STATEMENT OF POWER FLOW PROBLEM

Quantities associated with each bus in a system

Each bus in a power system is associated with four quantities and they are
real power (P), reactive power (Q), magnitude of voltage (V), and phase angle
of voltage (δ).
Work involved (or) to be performed by a load flow study
(i). Representation of the system by a single line diagram

(ii). Determining the impedance diagram using the information in single line
diagram (iii). Formulation of network equation

(iv). Solution of network equations

Iterative methods to solve load flow problems

The load flow equations are non linear algebraic equations and so explicit
solution as not possible. The solution of non linear equations can be obtained
only by iterative numerical techniques.

Mainly used for solution of load flow study


The Gauss seidal method, Newton Raphson method and Fast decouple
methods.

Flat voltage start

In iterative method of load flow solution, the initial voltages of all buses except
slack bus assumed as 1+j0 p.u. This is referred to as flat voltage start

Classification of Buses
Bus: The meeting point of various components in a power system is called a bus. - Different types of
buses in a power system

CLASSIFICATION OF BUSES

Bus

The meeting point of various components in a power system is called a bus.


The bus is a conductor made of copper or aluminum having negligible
resistance .At some of the buses power is being injected into the network,
whereas at other buses it is being tapped by the system lods.

Bus admittance matrix

The matrix consisting of the self and mutual admittance of the network of the
power system is called bus admittance matrix (Ybus).

Methods available for forming bus admittance matrix


Direct inspection method.
Singular transformation method.(Primitive network)

Different types of buses in a power system


Need for slack bus

The slack bus is needed to account for transmission line losses. In a power
system the total power generated will be equal to sum of power consumed by
loads and losses. In a power system only the generated power and load power
are specified for buses. The slack bus is assumed to generate the power
required for losses. Since the losses are unknown the real and reactive power
are not specified for slack bus.

Effect of acceleration factor in load flow study

Acceleration factor is used in gauss seidal method of load flow solution to


increase the rate of convergence. Best value of A.F=1.6

Generator buses are treated as load bus

If the reactive power constraint of a generator bus violates the specified limits
then the generator is treated as load bus.

Iterative Solution Using Gauss-Seidel Method -


Algorithm
Algorithm of Gauss seidal method - Calculations are simple and so the programming task is lessees. The
memory requirement is less. Useful for small systems;
ITERATIVE SOLUTION USING GAUSS-SEIDEL METHOD -
ALGORITHM
Algorithm of Gauss seidal method

Step1: Assume all bus voltage be 1+ j0 except slack bus. The voltage of the
slack bus is a constant voltage and it is not modified at any iteration

Step 2: Assume a suitable value for specified change in bus voltage which is
used to compare the actual change in bus voltage between K th and
(K+1) th iteration

Step 3: Set iteration count K = 0 and the corresponding voltages are V 10, V20,
V30, …… Vn0 except slack bus
Step 4: Set bus count P = 1

Step 5: Check for slack bus. It is a slack bus then goes to step 12 otherwise go
to next step

Step 6: Check for generator bus. If it is a generator bus go to next step.


Otherwise go to step 9

Step 7: Set │VPK│= │VP│ specified and phase of │VPK│ as the K th iteration
value if the bus P is a generator bus where │VP│ specified is the specified
magnitude of voltage for bus P. Calculate reactive power rating

Step 8: If calculated reactive power is within the specified limits then


consider the bus as generator bus and then set QP = QP K+1 Cal for this
iteration go to step 10
Step 9 : If the calculated reactive power violates the specified limit for
reactive power then treat this bus as load bus
If QP K+1 Cal < QP min then QP = QP min
QPK+1Cal > QP max then QP = QP max
Step10: For generator bus the magnitude of voltage does not change and so
for all iterations the magnitude of bus voltage is the specified value. The phase
of the bus voltage can be calculated using

Step 16: Calculate the line flows and slack bus power by using the bus
voltages

Gauss - Seidal method flow chart


Advantages and disadvantages of Gauss-Seidel method

Advantages: Calculations are simple and so the programming task is lessees.


The memory requirement is less. Useful for small systems;

Disadvantages: Requires large no. of iterations to reach converge .Not


suitable for large systems. Convergence time increases with size of the
system
Iterative Solution Using Newton-Raphson Method -
Algorithm
Iterative solution using Newton-Raphson method – Flow chart - Faster, more reliable and results are
accurate, require less number of iterations;

ITERATIVE SOLUTION USING NEWTON-RAPHSON


METHOD – ALGORITHM

Step 1: Assume a suitable solution for all buses except the slack bus. Let V p =
a+j0 for P
= 2,3,……n V1 = a+j0
Step 2 : Set the convergence criterion = ε0

Step 3 : Set iteration count K= 0

Step 4 : Set bus count P = 2

Step 5 : Calculate Pp and Qp using n

Step 6 : Evaluate ΔPPK = Pspec - PPK


Step 7 : Check if the bus is the question is a PV bus. If yes compare Q PK with
the limits. If it exceeds the limit fix the Q value to the corresponding limit and
treat the bus as PQ for that iteration and go to next step (or) if the lower limit
is not violated evaluate │ΔVP│2 = │ Vspec│2 - │VPK│2 and go to step 9

Step 8: Evaluate ΔQPK = Qspec - QPK

Step 9 : Advance bus count P = P+1 and check if all buses taken in to account
if not go to step 5

Step 10 : Determine the largest value of │ΔVP│2

Step 11: If ΔVP < ε go to step 16

Step 12: Evaluate the element of Jacobin matrices J 1, J2, J3, J4, J5 and J6

Step 13: Calculate ΔePK and ΔfPK

Step 14: Calculate ePK+1 = ePK + ΔePK and fPK+1 = fPK + ΔfPK

Step 15 : Advance count (iteration) K=K+1 and go to step 4

Step 16: Evaluate bus and line power and print the result

Iterative solution using Newton-Raphson method – Flow chart


Advantages and disadvantages of N.R method

Advantages: Faster, more reliable and results are accurate, require


less number of iterations;

Disadvantages: Program is more complex, memory is more complex.

• Prev Page

Iterative Solution Using Fast Decoupled Load Flow


Method - Algorithm
Step 1: Assume a suitable solution for all buses except the slack bus. Let Vp =1+j0 for P=2,3,...................n
and V=a+j0

ITERATIVE SOLUTION USING FAST DECOUPLED LOAD


FLOW METHOD - ALGORITHM

Step 1: Assume a suitable solution for all buses except the slack bus. Let Vp
=1+j0 for P=2,3,...................n and V=a+j0

Step2: Set the convergence criterion = ε0

Step3: Set iteration count K = 0


Step 4: Set bus count P = 2

Step 5: Calculate Pp and Qp using

Step 6: Compute the real and reactive power mismatches ΔPK and ΔQK. If the
mismatches Are with in desirable tolerance the iteration end

Step 7: Normalize the mismatches by dividing each entry by its respective bus
voltage

magnitude ΔPK =ΔP 2K / V 2K


Step 8: Solve for the voltage magnitude and the correction factors ΔVK and δK
by using the constant matrices B’ and B” which are extracted from the bus
admittance matrix Y Bus

[B’] δK = ΔPK

[B”]ΔQK = ΔQK

Step 9: Up date the voltage magnitude and angel vectors

δK+1 = δK +ΔδK

VK+1 = VK +ΔVK
Step 10: Check if all the buses are taken into account if yes go to next step
otherwise go to next step. Otherwise go to step 4

Step 11: Advance iteration count K = K+1 go to step 3

Step 12: Evaluate bus and load powers and print the results
Compare the Gauss Seidel and Newton Raphson
Methods of Load Flow Study

Y matrix of the sample power system as shown in fig. Data for this system is given in table.

COMPARE THE GAUSS SEIDEL AND NEWTON RAPHSON


METHODS OF LOAD FLOW STUDY
Y matrix of the sample power system as shown in fig. Data for this system is
given in table.

Advantages and disadvantages of Gauss-Seidel method

Advantages: Calculations are simple and so the programming task is lessees.


The memory requirement is less. Useful for small systems;
Disadvantages: Requires large no. of iterations to reach converge .Not
suitable for large systems. Convergence time increases with size of the system

Advantages and disadvantages of N.R method

Advantages: Faster, more reliable and results are accurate, require less
number of iterations; Disadvantages: Program is more complex, memory is
more complex.

Important Short Questions and Answers:


Power Flow Analysis
Power System Analysis - Power Flow Analysis - Important Short Questions and Answers: Power
Flow Analysis

1. What is power flow study?

The study of various methods of solution to power system network is


reffered to as power flow or load flow study.

2. What are the informations that are obtained from a load flow study?

The information obtained from a load flow study are magnitude and phase
of bus voltages, real and reactive power flowing in each line and the line losses. The
load flow solution also gives the initial conditions of the system when the transient
behaviour of the system is to be studied.

3. What is the need for load flow study?

The load flow study of a power system is essential to decide the best
operation of existing system and for planning the future expansion of the system.
Also essential for designing a new system.

Ø What are the works involved in a load flow study? (i)Representation of the
system by single line diagram.

(ii)Determining the impedance diagram using the information in single line


diagram. (iii)Formulation of network equations.

(iv)Solution of network equations.

Ø What are the quantities that are associated with each bus in a system?

Each bus in a system are associated with four quantities and they are real
power,reactive power, magnitude of voltage and phase angle of voltage.
Ø What are the different types of buses? (i)Load bus or PQ bus

(ii)Generator bus or voltage controlled bus or PV bus (iii)Slack bus or Swing


bus or Reference bus.

ØDefine voltage controlled bus.

A bus is called voltage controlled bus if the magnitude of voltage and real
power are specified. The magnitude of voltage is not allowed to change.

8. What is PQ bus?

A bus is called PQ are load bus when real and reactive components of power
are specified for the bus. In a load bus the voltage is allowed to vary within the
permissible value.

9. What is swing bus?


A bus is called Swing bus when the magnitude and phase of the voltage are
specified for it. Swing bus is the reference bus for load flow solution and it is
required for accounting line losses.

10. What is the need for slack bus?

The slack bus is needed to account for transmission losses.

Ø What are the iterative methods used for solution of load flow problems? Guass-
Seidal (GS) method and Newton Raphson(NR) method.

Ø Why do we go for iterative methods to solve load problems?

The load flow equations are nonlinear algebraic equations and so explicit
solution is not possible. The solution of nonlinear equations can be obtained only
by iterative numerical techniques.

13. When the generator bus is treated as load bus?

If the reactive power of a generator bus violates the specified limits then the
generator bus is treated as load bus.
14. What are the advantages of GS method?

(i)Calculations are simple and so the programmimg task is lesser.

(ii)The memory requirement is less.

(iii)Useful for small systems.

15. What are the disadvantages of GS method?

(i)Requires large number of iterations to reach convergence.

(ii) Not suitable for large systems.

(iii) Convergence time increases with size of the system.

16. What are the advantages of NR method?

(i)Faster, more reliable and the results are accurate.


(ii) Requires less number of iterations to reach convergence.

(iii)The numbers of iterations are independent of the size of the system.

(iv) Suitable for large systems.

17. What are the disadvantages of NR method?

(i)The programming is more complex.

(ii)Memory requirement is more.

(iii)Computation time per iteration is higher due to large number of calculations per
iterations.

Importance Short Circuit (Or) For Fault Analysis


A fault in a circuit is any failure which interferes with the normal flow of current. The faults are
associated with abnormal change in current, voltage and frequency of the power system.

IMPORTANCE SHORT CIRCUIT (OR) FOR FAULT


ANALYSIS

Fault

A fault in a circuit is any failure which interferes with the normal flow of
current. The faults are associated with abnormal change in current, voltage
and frequency of the power system.
Faults occur in a power system
The faults occur in a power system due to
(i). Insulation failure of equipment

(ii). Flashover of lines initiated by a lighting stroke

(iii). Due to permanent damage to conductors and towers or due to accidental


faulty operations.

Various types of faults


(i) Series fault or open circuit fault

One open conductor fault


Two open conductor fault

(ii) Shunt fault or short circuit fault. Symmetrical fault or balanced fault
▪ Three phase fault

Unsymmetrical fault or unbalanced fault


▪ Line to ground (L-G) fault
▪ Line to Line (L-L) fault
▪ Double line to ground (L-L-G) fault

Relative frequency of occurrence of various types of fault


Symmetrical fault or balanced three phase fault

This type of fault is defined as the simultaneous short circuit across all the
three phases. It occurs infrequently, but it is the most severe type of fault
encountered. Because the network is balanced, it is solved by per phase basis
using Thevenins theorem or bus impedance matrix or KVL, KCL laws.

Basic Assumptions in Fault Analysis of Power Systems


Representing each machine by a constant voltage source behind proper reactance which may be X”, X’,
or X

BASIC ASSUMPTIONS IN FAULT ANALYSIS OF POWER


SYSTEMS

(i). Representing each machine by a constant voltage source behind proper


reactance which may be X”, X’, or X

(ii). Pre-fault load current are neglected


(iii). Transformer taps are assumed to be nominal

(iv). Shunt elements in the transformers model that account for magnetizing
current and core loss are neglected

(v). A symmetric three phase power system is conducted

(vi). Shunt capacitance and series resistance in transmission are neglected

(vii). The negative sequence impedances of alternators are assumed to be the


same as their positive sequence impedance Z+ = Z-

Need for short circuit studies or fault analysis

Short circuit studies are essential in order to design or develop the protective
schemes for various parts of the system .To estimate the magnitude of fault
current for the proper choice of circuit breaker and protective relays.

Bolted fault or solid fault

A Fault represents a structural network change equivalent with that caused


by the addition of impedance at the place of a fault. If the fault impedance is
zero, the fault is referred as bolted fault or solid fault.

Reason for transients during short circuits


The faults or short circuits are associated with sudden change in currents.
Most of the components of the power system have inductive property which
opposes any sudden change in currents, so the faults are associated with
transients.

Doubling effect

If a symmetrical fault occurs when the voltage wave is going through zero then
the maximum momentary short circuit current will be double the value of
maximum symmetrical short circuit current. This effect is called doubling
effect.

DC off set current


The unidirectional transient component of short circuit current is called DC
off set current.

• Prev Page

Symmetrical Fault
In symmetrical faults all the three phases are short circuited to each other and to earth also.

SYMMETRICAL FAULT

In symmetrical faults all the three phases are short circuited to each other and
to earth also. Such faults are balanced and symmetrical in the sense that the
voltage and current of the system remains balanced even after the fault and it
is enough if we consider any one phase

Short circuit capacity of power system or fault level.


Short circuit capacity (SCC) or Short circuit MVA or fault level at a bus is
defined as the product of the magnitude of the pre fault bus voltage and the
post fault current

Synchronous reactance or steady state condition reactance

The synchronous reactance is the ratio of induced emf and the steady state
rms current. It is the sum of leakage reactance (Xl) and the armature reactance
(Xa).

Sub transient reactance

The synchronous reactance is the ratio of induced emf on no load and the sub
transient symmetrical rms current.
Transient reactance

The synchronous reactance is the ratio of induced emf on no load and the
transient symmetrical rms current.
Fault current in fig., if the Pre-fault voltage at the fault point is 0.97 p.u.

Thevenin’s theorem:
(i). Fault current = Eth / (Zth+Zf)
(ii). Determine current contributed by the two generators IG 1 = If *
(Z2/(Z1+Z2))
IG2 = If * (Z1 / (Z1+Z2))
(iii). Determine Post fault voltage Vif = Vi°+ΔV = V°+(-Zi2*IGi)
(iv). Determine post fault voltage line flows Iij = (Vi –Vj) / Zij series
(v). Short circuit capacity If = │Eth│2 / Xth

• Prev Page

Symmetrical Fault
In symmetrical faults all the three phases are short circuited to each other and to earth also.

SYMMETRICAL FAULT

In symmetrical faults all the three phases are short circuited to each other and
to earth also. Such faults are balanced and symmetrical in the sense that the
voltage and current of the system remains balanced even after the fault and it
is enough if we consider any one phase

Short circuit capacity of power system or fault level.

Short circuit capacity (SCC) or Short circuit MVA or fault level at a bus is
defined as the product of the magnitude of the pre fault bus voltage and the
post fault current

Synchronous reactance or steady state condition reactance


The synchronous reactance is the ratio of induced emf and the steady state
rms current. It is the sum of leakage reactance (Xl) and the armature reactance
(Xa).

Sub transient reactance

The synchronous reactance is the ratio of induced emf on no load and the sub
transient symmetrical rms current.
Transient reactance

The synchronous reactance is the ratio of induced emf on no load and the
transient symmetrical rms current.

Fault current in fig., if the Pre-fault voltage at the fault point is 0.97 p.u.

Thevenin’s theorem:
(i). Fault current = Eth / (Zth+Zf)
(ii). Determine current contributed by the two generators IG 1 = If *
(Z2/(Z1+Z2))
IG2 = If * (Z1 / (Z1+Z2))
(iii). Determine Post fault voltage Vif = Vi°+ΔV = V°+(-Zi2*IGi)
(iv). Determine post fault voltage line flows Iij = (Vi –Vj) / Zij series
(v). Short circuit capacity If = │Eth│2 / Xth

• Prev Page

Solved problems: Fault Analysis - Balanced Faults


Power System Analysis - Fault Analysis - Balanced Faults - Solved problems: Fault Analysis - Balanced
Faults

SOLVED PROBLEMS
Problem 1

A synchronous generator and a synchronous motor each rated 20MVA,


12.66KV having 15% reactance are connected through transformers and a
line as shown in fig. the transformers are rated 20MVA,12.66/66KV and
66/12.66KV with leakage reactance of 10% each. The line has a reactance of
8% on base of 20MVA, 66 KV. The motor is drawing 10MW at 0.8 leading
power factors and a terminal voltage 11KV when symmetrical three phase
fault occurs at the motors terminals. Determine the generator and motor
currents. Also determine the fault current.
Two generators G1 and G2 are rated 15MVA, 11KV and 10MVA, 11KV
respectively. The generators are connected to a transformer as shown in fig.
Calculate the sub transient current in each generator when a three phase fault
occurs on the high voltage side of the transformer.

A radial power system network is shown in fig. a three phase balanced fault
occurs at F. Determine the fault current and the line voltage at 11.8 KV bus
under fault condition.
Problem : 2

A 100MVA,11KV generator with X’’=0.20 p.u is connected through a


transformer and line to a bus bar that supplies three identical motor as shown
in fig. and each motor has X’’=0.20 p.u and X’=0.25 p.u on a base of
20MVA,33KV.the bus voltage at the motors is 33KV when a three phase
balanced fault occurs at the point F. Calculate

(a) subtransient current in the fault


(b) subtransient current in the circuit breaker B
(c) Momentary current in the circuit breaker B
(d) The current to be interrupted by CB B in (i) 2 cycles (ii) 3 cycles (iii) 5
cycles (iv) 8 cycles
Obtain impedance matrix ZBUS for shown in figure.
• Prev Page
Important Short Questions and Answers:
Fault Analysis - Balanced Faults
Power System Analysis - Fault Analysis - Balanced Faults - Important Short Questions and Answers:
Fault Analysis - Balanced Faults

1. What is power flow study?

The study of various methods of solution to power system network


is reffered to as power flow or load flow study.

2. What are the informations that are obtained from a load flow study?

The information obtained from a load flow study are magnitude


and phase of bus voltages, real and reactive power flowing in each line
and the line losses. The load flow solution also gives the initial conditions
of the system when the transient behaviour of the system is to be
studied.
3. What is the need for load flow study?

The load flow study of a power system is essential to decide the


best operation of existing system and for planning the future expansion
of the system. Also essential for designing a new system.

4. What are the works involved in a load


flow study? (i)Representation of the
system by single line diagram.

(ii)Determining the impedance diagram using the


information in single line diagram. (iii)Formulation of
network equations.

(iv)Solution of network equations.


5. What are the quantities that are associated with each bus in a system?

Each bus in a system are associated with four quantities and they
are real power,reactive power, magnitude of voltage and phase angle of
voltage.

6. What are the different types of buses?

(i)Load bus or PQ bus

(ii)Generator bus or voltage controlled bus or PV bus

(iii)Slack bus or Swing bus or Reference bus.

7. Define voltage controlled bus.


A bus is called voltage controlled bus if the magnitude of voltage
and real power are specified. The magnitude of voltage is not allowed to
change.

8. What is PQ bus?

A bus is called PQ are load bus when real and reactive components
of power are specified for the bus. In a load bus the voltage is allowed to
vary within the permissible value.

9. What is swing bus?

A bus is called Swing bus when the magnitude and phase of the
voltage are specified for it. Swing bus is the reference bus for load flow
solution and it is required for accounting line losses.
10. What is the need for slack bus?

The slack bus is needed to account for transmission losses.

i. What are the iterative methods used for solution


of load flow problems? Guass-Seidal (GS)
method and Newton Raphson(NR) method.

ii. Why do we go for iterative methods to solve load problems?

The load flow equations are nonlinear algebraic equations and so


explicit solution is not possible. The solution of nonlinear equations can
be obtained only by iterative numerical techniques.

13. When the generator bus is treated as load bus?


If the reactive power of a generator bus violates the specified limits
then the generator bus is treated as load bus.

14. What are the advantages of GS method?

(i)Calculations are simple and so the


programmimg task is lesser. (ii)The memory
requirement is less.

(iii)Useful for small systems.

15. What are the disadvantages of GS method?

1. Requires large number of iterations to reach convergence.

2. Not suitable for large systems.


3. Convergence time increases with size of the system.

16. What are the advantages of NR method?

(i)Faster, more reliable and the results are accurate.

(ii) Requires less number of iterations to reach convergence.

(iii)The numbers of iterations are independent of the size of the system.

(iv) Suitable for large systems.

17. What are the disadvantages of NR method?

(i)The programming is more complex.

(ii)Memory requirement is more.


(iii)Computation time per iteration is higher due to large number of
calculations per iterations.

Introduction to Symmetrical Components


Symmetrical components of a 3 phase system

INTRODUCTION TO SYMMETRICAL COMPONENTS

Symmetrical components of a 3 phase system

In a 3 phase system, the unbalanced vectors (either currents or voltage) can


be resolved into three balanced system of vectors.

They are Positive sequence components Negative sequence components


Zero sequence components
Unsymmetrical fault analysis can be done by using symmetrical components.

Positive sequence components

It consists of three components of equal magnitude, displaced each other by


120˚ in phase and having the phase sequence abc .
Negative sequence components

It consists of three components of equal magnitude, displaced each other by


120˚ in phase and having the phase sequence acb .

Zero sequence components


It consists of three phasors equal in magnitude and with zero phase
displacement from each other.
Sequence operator

In unbalanced problem, to find the relationship between phase voltages and


phase currents, we use sequence operator ‘a’.
a = 1∠120˚ == - 0.5+j0.86

Unbalanced currents from symmetrical currents


Let, Ia, Ib, Ic be the unbalanced phase currents
Let, Ia0, Ia1, Ia2 be the symmetrical components of phase a

Determination of symmetrical currents from unbalanced currents.


Let, Ia, Ib, Ic be the unbalanced phase currents
Let, Ia0, Ia1, Ia2 be the symmetrical components of phase a
• Prev Page

Sequence Impedances Sequence Networks


The sequence impedances are the impedances offered by the power system components or elements to
+ve, -ve and zero sequence current.

SEQUENCE IMPEDANCES SEQUENCE NETWORKS

The sequence impedances are the impedances offered by the power system
components or elements to +ve, -ve and zero sequence current.

The single phase equivalent circuit of power system consisting of impedances


to current of any one sequence only is called sequence network.

The phase voltage across a certain load are given as

Compute positive, negative and zero sequence component of voltage


A balanced delta connected load is connected to a three phase system and
supplied to it is a current of 15 amps. If the fuse is one of the lines melts,
compute the symmetrical components of line currents.
Draw zero sequence network of the power system as shown in fig.

Draw zero sequence network of the power system as shown in fig.


Draw zero sequence network of the power system as shown in fig. Data are
given below.
Representation of Single Line to Ground, Line to Line
and Double Line to Ground Fault Conditions
A 50MVA, 11KV, synchronous generator has a sub transient reactance of 20%.

REPRESENTATION OF SINGLE LINE TO GROUND, LINE TO LINE AND


DOUBLE LINE TO GROUND FAULT CONDITIONS.

A 50MVA, 11KV, synchronous generator has a sub transient reactance of 20%.


The generator supplies two motors over a transmission line with
transformers at both ends as shown in fig. The motors have rated inputs of 30
and 15 MVA, both 10KV, with 25% sub transient reactance. The three phase
transformers are both rated 60MVA, 10.8/121KV, with leakage reactance of
10% each. Assume zero sequence reactance for the generator and motors of
6% each. Current limiting reactors of 2.5 ohms each are connected in the
neutral of the generator and motor number 2. The zero sequence reactance of
the transmission line is 300 ohms. The series reactance of the line is 100
ohms. Draw the positive, negative and zero sequence networks.
Unbalanced Fault Analysis Problem Formulation
A 30 MVA, 13.2KV synchronous generator has a solidly grounded neutral. Its positive, negative and zero
sequence impedances are 0.30, 0.40 and 0.05 p.u respectively.

Unbalanced Fault Analysis Problem Formulation

A 30 MVA, 13.2KV synchronous generator has a solidly grounded neutral. Its


positive, negative and zero sequence impedances are 0.30, 0.40 and 0.05 p.u
respectively. Determine the following:
a) What value of reactance must be placed in the generator neutral so that
the fault current for a line to ground fault of zero fault impedance shall not
exceed the rated line current?

b) What value of resistance in the neutral will serve the same purpose?

c) What value of reactance must be placed in the neutral of the generator to


restrict the fault current to ground to rated line current for a double line to
ground fault?

d) What will be the magnitudes of the line currents when the ground
current is restricted as above?

e) As the reactance in the neutral is indefinitely increased, what are the


limiting values of the line currents?
Two alternators are operating in parallel and supplying a synchronous motor
which is receiving 60MW power at 0.8 power factor lagging at 6.0 KV. Single
line diagram for this system is given in fig. Data are given below. Compute the
fault current when a single line to ground fault occurs at the middle of the line
through a fault resistance of 4.033 ohm.
• Prev Page

Important Short Questions and Answers:


Symmetrical Components And Unbalanced
Fault Analysis
Power System Analysis - Symmetrical Components And Unbalanced Fault Analysis - Important
Short Questions and Answers: Symmetrical Components And Unbalanced Fault Analysis
1. What is meant by a fault?

A fault in a circuit is any failure which interferes with the normal flow of
current. The faults are associated with abnormal change in current, voltage and
frequency of the power system.

2. Why fault occur in a power system?

The faults occur in a power system due to insulation failure of


equipments,flashover of lines initiated by a lightning stroke, due to permanent
damage to conductors and towers or due to accidental faulty operations.

3. List the various types of shunt and series faults. The various types of shunt faults
are:

(i)Line to Ground fault

(ii) Line to Line fault

(iii)Double line to Ground fault

(iv) Three phase fault


The various types of series faults are:

(i)One open conductor fault

(ii) Two open conductor fault

4. What is symmetrical unsymmetrical fault?

The fault is called symmetrical fault if the fault current is equal in all the
phases.The fault is called unsymmetrical fault if the fault current is not equal in all
the phases.

5. Name any two methods of reducing short –circuit current.

(i)By providing neutral reactance.

(ii)By introducing a large value of shunt reactance between buses.

6. What is meant by fault calculations?


The fault condition of a power system can be divided into subtransient,
transient and steady state periods. The currents in the various parts of the system
and in the fault are different in these periods. The estimation of these currents for
various types of faults at various locations in the system are commonly referred to
as fault calculations.

7. What is the need for short circuit studies or fault analysis?

The short circuit studies are essential in order to design or develop the
protective schemes for various parts of the system. The protective schemes
consists of current and voltage sensing devices , protective relays and circuit
breakers. The selection of these devices mainly depends on various currents that
may flow in the fault conditions.

8. What is synchronous reactance?

The synchronous reactance is the ratio of induced emf and the steady rms
current. It is the sum of leakage reactance and the reactance representing armature
reaction.

9. Define subtransient reactance.


The subtransient reactance is the ratio of induced emf on no load and the
subtransient

symmetrical rms current.

10. Define transient reactance.

The subtransient reactance is the ratio of induced emf on no load and the
transient symmetrical rms current.

11. Name the fault in which positive, negative and zero sequence component
currents are equal.

In Single line to ground fault positive, negative and zero sequence


component currents are equal.

12. Name the fault in which positive and negative sequence component currents
together is equal to zero sequence current in magnitude.

Double line to ground fault.


13. Define positive sequence impedance.

The positive sequence impedance of an equipment is the impedance


offered by the equipment to the flow of positive sequence currents.

14. Define negative sequence impedance.

The negative sequence impedance of an equipment is the impedance


offered by the equipment to the flow of negative sequence currents.

15. Write the boundary condition in single line to ground fault.

Va = 0 ; I b = I c = 0

16. What are the boundary conditions in line to line fault?

I a = 0 ; I b + I c = 0 ; V b = Vc

17. Write down the boundary condition in double line to ground fault. I a = 0 ; Vb = 0
; Vc= 0

18. Give the boundary condition for 3-phase fault.


Ia + Ib = Ic = 0 ; Va = Vb = Vc= 0

Part-B

1.A balanced delta connected load is connected to a three phase system and
supplied to it is a current of 15 amps. If the fuse is one of the lines melts, compute
the symmetrical components of line currents

2. 2.Draw zero sequence network of the power system as shown in fig.

3.A 50MVA, 11KV, synchronous generator has a sub transient reactance of 20%.The
generator supplies two motors over a transmission line with transformers at both
ends as shown in fig. The motors have rated inputs of 30 and 15 MVA, both 10KV,
with 25% sub transient reactance. The three phase transformers are both rated
60MVA, 10.8/121KV, with leakage reactance of 10% each. Assume zero sequence
reactance for the generator and motors of 6% each. Current limiting reactors of 2.5
ohms each are connected in the neutral of the generator and motor number 2. The
zero sequence reactance of the transmission line is 300 ohms. The series reactance
of the line is 100 ohms. Draw the positive, negative and zero sequence networks.

4.A 30 MVA, 13.2KV synchronous generator has a solidly grounded neutral. Its
positive, negative and zero sequence impedances are 0.30, 0.40 and 0.05 p.u
respectively. Determine the following:

i. What value of reactance must be placed in the generator neutral so that the fault
current for a line to ground fault of zero fault impedance shall not exceed the rated
line current?

ii. What value of resistance in the neutral will serve the same purpose?

iii. What value of reactance must be placed in the neutral of the generator to
restrict the fault current to ground to rated line current for a double line to ground
fault?
iv. What will be the magnitudes of the line currents when the ground current is
restricted as above?

v. j) As the reactance in the neutral is indefinitely increased, what are the


limiting values of the line currents?

Classification of Power System Stability - Angle and


Voltage Stability
Importance Of Stability Analysis In Power System Planning And Operation

Importance Of Stability Analysis In Power System


Planning And Operation

Power system stability

The stability of an interconnected power system means is the ability of the


power system is to return or regain to normal or stable operating condition
after having been subjected to some form of disturbance.

Classification Of Power System Stability - Angle And Voltage


Stability

Power system stability is classified


ANGLE AND VOLTAGE STABILITY

Rotor angle stability

Rotor angle stability is the ability of interconnected synchronous machines of


a power system to remain in synchronism.

Steady state stability

Steady state stability is defined as the ability of the power system to bring it
to a stable condition or remain in synchronism after a small disturbance.
Steady state stability limit

The steady sate stability limit is the maximum power that can be transferred
by a machine to receiving system without loss of synchronism

Transient stability

Transient stability is defined as the ability of the power system to bring it to a


stable condition or remain in synchronism after a large disturbance.

Transient stability limit

The transient stability limit is the maximum power that can be transferred by
a machine to a fault or a receiving system during a transient state without loss
of synchronism.Transient stability limit is always less than steady state
stability limit

Dynamic stability

It is the ability of a power system to remain in synchronism after the initial


swing (transient stability period) until the system has settled down to the new
steady state equilibrium condition

Voltage stability

It is the ability of a power system to maintain steady acceptable voltages at all


buses in the system under normal operating conditions and after being
subjected to a disturbance.
Causes of voltage instability

A system enters a state of voltage instability when a disturbance, increase in


load demand, or change in system condition causes a progressive and
uncontrollable drop in voltage. The main factor causing instability is the
inability of the power system to meet the demand for reactive power.

• Prev Page

Power angle equation and draw the power angle curve


Power System Analysis - Stability Analysis - Power angle equation and draw the power angle curve

Power angle equation and draw the power angle curve

Where, P – Real Power in watts

Vs – Sending end voltage;

Vr- Receiving end voltage

XT - Total reactance between sending end receiving end

δ - Rotor angle.
• Prev Page

Rating of Circuit Breaker


The rating of a circuit breaker includes, 1) Rated short circuit breaking current. 2) Rated short circuit
making current. 3) Rated operating sequence of circuit breaker. 4) Rated short time current.

Rating of Circuit Breaker

The rating of a circuit breaker includes,

1) Rated short circuit breaking current.

2) Rated short circuit making current.


3) Rated operating sequence of circuit breaker.

4) Rated short time current.

Short circuit breaking current of circuit breaker

This is the maximum short circuit current which a circuit breaker can withstand
before it. Finally cleared by opening its contacts. When a short circuit flows
through a circuit breaker, there would be thermal and mechanical stresses in
the current carrying parts of the breaker. If the contact area and cross-section
of the conducting parts of the circuit breaker are not sufficiently large, there
may be a chance of permanent damage in insulation as well as conducting parts
of the CB. The short circuit current has a certain value at the instant of contact
separation. The breaking current refers to value of current at the instant of the
contact separation. The rated values of transient recovery voltage are specified
for various rated voltage of circuit breakers. For specified conditions of rated
TRV and rated power frequency recovery voltage, a circuit breaker has a certain
limit of breaking current. This limit is determined by conducting short circuit
type tests on the circuit breaker. The waveforms of short circuit current are
obtained during the breaking test. The evaluation of the breaking current is
explained in Fig. 3. The breaking current is expressed by two values. The r.m.s
values of a.c. components are expressed in KA. the standard values being 8, 10,
12.5, 16, 20, 25, 31.5, 40, 45, 63, 80 and 100KA. The earlier practice was to
express the rated breaking capacity of a circuit breaker in terms of MVA given
as follows Rated Breaking MVA capacity = √3 x KV x KA Where MVA = Breaking
capacity of a circuit breaker kV KV = Rated voltage KA = Rated breaking current.

This practice of specifying the breaking capacity in terms of MVA is convenient


while calculating the fault levels. However, as per the revised standards, the
breaking capacity is expressed in KA for specified conditions of TRV and this
method takes into account both breaking current and TRV. The breaking
capacity can be both symmetrical and asymmetrical in nature. In asymmetrical
breaking capacity the DC component of the current is added. While selecting
the circuit breaker for a particular location in the power system the fault level
at that location is determined. The rated breaking current can then be selected
from standard range.
Rated short circuit making capacity

The short circuit making capacity of circuit breaker is expressed in peak value
not in rms value like breaking capacity.It may so happen that circuit breaker
may close on an existing fault. In such cases the current increase to the
maximum value at the peak of first current loop. The circuit breaker should be
able to close without hesitation as contact touch. The circuit breaker should be
able to withstand the high mechanical forces during such a closure. These
capabilities are proved by carrying out making current test. The rated short
circuit making current of a circuit breaker is the peak value of first current loop
of short circuit current (I pk)Which the circuit breaker is capable of making at
its rated voltage. The rated short circuit making current should be least 2.5
times the r.m.s. value of a.c. component of rated breaking current . Rated
making current = 1.8 x √2 x Rated short circuit breaking = 2.5 x Rated short
circuit breaking current In the above equation the factor √2 convert the r.m.s
value to peak value. Factor 1.8 takes into account the doubling effect of short
circuit current with consideration to slight drop in current during the first
quarter cycle .

Rated operating sequence or duty cycle of circuit breaker

This is mechanical duty requirement of circuit breaker operating mechanism.


The sequence of rated operating duty of a circuit breaker has been specified as
O – t – CO – t‟ – CO Where O indicates opening operation of the CB. CO
represents closing operation immediately followed by an opening operation
without any intentional time delay. t‟ is time between two operations which is
necessary to restore the initial conditions and / or to prevent undue heating of
conducting parts of circuit breaker. t = 0.3 sec for circuit breaker intended for
first auto re closing duty, if not otherwise specified. Suppose rated duty circle
of a circuit breaker is 0 – 0.3 sec – CO – 3 min – CO. This means, an opening
operation of circuit breaker is followed by a closing operation after a time
interval of 0.3 sec, then the circuit breaker again opens without any intentional
time delay. After this opening operation the CB is again closed after 3 minutes
and then instantly trips without any intentional time delay.
Rated short time current

This is the current limit which a circuit breaker can carry safely for certain
specific time without any damage.

The circuit breakers do not clear the short circuit current as soon as any fault
occurs in the system. There always some intentional and an intentional time
delays present between the instant of occurrence of fault and instant of clearing
the fault by CB. This delay is present because of time of operation of protection
relays, time of operation of circuit breaker and also there may be some
intentional time delay imposed in relay for proper coordination of power
system protection. Hence, after fault, a circuit breaker has to carry the short
circuit for certain time. The summation of all time delays should not be more
than 3 seconds, hence a circuit breaker should be capable of carrying a
maximum fault current for at least this short period of time.

The short circuit current may have two major affects inside a circuit breaker.

1. Because of the high electric current, there may be high thermal stress
in the insulation and conducting parts of CB.

2. The high short circuit current, produces significant mechanical


stresses in differentcurrent carrying parts of the circuit breaker.

A circuit breaker is designed to withstand these stresses. But no circuit breaker


has to carry a short circuit current not more than a short period depending
upon the coordination of protection. So it is sufficient to make CB capable of
withstanding affects of short circuit current for a specified short period.

The rated short time current of a circuit breaker is at least equal to rated short
circuit breaking current of the circuit breaker.
Rated voltage of circuit breaker

Rated voltage of circuit breaker depends upon its insulation system. For below
400 KV system, the circuit breaker is designed to withstand 10% above the
normal system voltage. For above or equal 400 KV system the insulation of
circuit breaker should be capable of withstanding 5% above the normal system
voltage. That means, rated voltage of circuit breaker corresponds to the highest
system voltage. This is because during no load or small load condition the
voltage level of power system is allowed rise up to highest voltage rating of the
system.

A circuit breaker is also subject to two other high voltage condition.

1) Sudden disconnection of huge load for any other cause, the voltage
imposed on the CB and also between the contacts when the CB is open, may be
very high compared to higher system voltage. This voltage may be of power
frequency but does not stay for very long period as this high voltage situation
must be cleared by protective switchgear. But a circuit breaker may have to
withstand this power frequency over voltage, during its normal life span.

The Circuit Breaker must be rated for power frequencies withstand voltage for
a specific time only. Generally the time is 60 seconds. Making power frequency
withstand capacity, more than 60 second is not economical and not practically
desired as all the abnormal situations of electrical power system are definitely
cleared within much smaller period than 60 seconds.

2) Like other apparatuses connected to power system, a circuit breaker


may have also to face lightening impulse and switching impulses during its life
span.
The insulation system of CB has to withstand these impulse voltage waveform.
So a circuit breaker is designed to withstand this impulse peaky voltage for
microsecond range only.

• Prev Page

Air blast circuit breaker: Working principle, Advantages,


Disadvantages
This type of circuit breakers, is those kind of circuit breaker which operates in air at atmospheric
pressure. After development of oil circuit breaker, the medium voltage air circuit breaker (ACB) is
replaced completely by oil circuit breaker in different countries.

Air blast circuit breaker


This type of circuit breakers, is those kind of circuit breaker which operates in
air at atmospheric pressure. After development of oil circuit breaker, the
medium voltage air circuit breaker (ACB) is replaced completely by oil circuit
breaker in different countries. But in countries like France and Italy, ACBs are
still preferable choice up to voltage 15 KV. It is also good choice to avoid the
risk of oil fire, in case of oil circuit breaker. In America ACBs were exclusively
used for the system up to 15 KV until the development of new vacuum and SF6
circuit breakers.

Working principle of air circuit breaker(ACB)

The working principle of this breaker is rather different from those in any other
types of circuit breakers. The main aim of all kind of circuit breaker is to prevent
the reestablishment of arcing after current zero by creating a situation where
in the contact gap will withstand the system recovery voltage. The air circuit
breaker does the same but in different manner. For interrupting arc it creates
an arc voltage in excess of the supply voltage. Arc voltage is defined as the
minimum voltage required maintaining the arc. This circuit breaker increases
the arc voltage by mainly three different ways,

1. It may increase the arc voltage by cooling the arc plasma. As the temperature
of arc plasma is decreased, the mobility of the particle in arc plasma is reduced,
hence more voltage gradient is required to maintain the arc.

2. It may increase the arc voltage by lengthening the arc path. As the length of
arc path is increased, the resistance of the path is increased, and hence to
maintain the same arc current more voltage is required to be applied across the
arc path. That means arc voltage is increased.

3. Splitting up the arc into a number of series arcs also increases the arc voltage.
The first objective is usually achieved by forcing the arc into contact with as
large an area as possible of insulating material. Every air circuit breaker is fitted
with a chamber surrounding the contact. This chamber is called „arc chute‟. The
arc is driven into it. If inside of the arc chute is suitably shaped, and if the arc
can conform to the shape, the arc chute wall will help to achieve cooling. This
type of arc chute should be made from some kind of refractory material

The second objective that is lengthening the arc path is achieved concurrently
with the first objective. If the inner walls of the arc chute is shaped in such a
way that the arc is not only forced into close proximity with it but also driven
into a serpentine channel projected on the arc chute wall. The lengthening of
the arc path increases the arc resistance.

The third objective is achieved by using metal arc slitter inside the arc chute.
The main arc chute is divided into numbers of small compartments by using
metallic separation plates. These metallic separation plates are actually the arc
splitters and each of the small compartments behaves as individual mini arc
chute. In this system the initial arc is split into a number of series arcs, each of
which will have its own mini arc chute.
1. Main contacts
2. Arcing contacts
3. Arc rifling in the direction of the arrow
4. Arcsplitterplates

5.Current carrying terminals

6. Arc runners Arc getting split


In the air reservoir there is a high pressure air stored between 20 to 30 kg/cm2.
And that air is taken from compressed air system. On the reservoir there are
three hollow insulator columns mounted with valves at their base. On the top
of the hollow insulator chambers there are double arc extinguishing chambers
mounted. The current carrying parts connect the three arc extinction chambers
to each other in series and the pole to the neighboring equipment, since there
exist a very high voltage between the conductor and the air reservoir, the entire
arc extinction chamber assembly is mounted on insulators. Since there are
three double arc extinction poles in series, there are six breakers per pole. Each
arc extinction chamber consists of one twin fixed contact. There are two moving
contacts. The moving contacts can move axially so as to open or close. Its
opening or closing mechanism depends on spring pressure and air pressure.

The operation mechanism operates the rods when it gets a pneumatic or


electrical signal. The valves open so as to send the high pressure air in the
hollow of the insulator. The high pressure air rapidly enters the double arc
extinction chamber. As the air enters into the arc extinction chamber the
pressure on the moving contacts becomes more than spring pressure and it
causes the contacts to be open.

The contacts travel through a short distance against the spring pressure. At the
end of contacts travel the part for outgoing air is closed by the moving contacts
and the entire arc extinction chamber is filled with high pressure air, as the air
is not allowed to go out. However, during the arcing period the air goes out
through the openings and takes away the ionized air While closing, the valve is
turned so as to close connection between the hollow of the insulator and the
reservoir.
The valve lets the air from the hollow insulator to the atmosphere. As a result
the pressure of air in the arc extinction chamber is dropped down to the
atmospheric pressure and the moving contacts close over the fixed contacts by
virtue of the spring pressure, the opening is fast because the air takes a
negligible time to travel from the reservoir to the moving contact. The arc is
extinguished within a cycle. Therefore, air blast circuit breaker is very fast in
breaking the current. Closing is also fast because the pressure in the arc
extinction chamber drops immediately as the value operates and the contacts
close by virtue of the spring pressure.

Advantages:

• How air blast circuit breaker is better than oil circuit breaker:

• The growth of dielectric strength is so rapid that final contact gap needed
for arc extinction is very small, this reduces the size of device.

• The risk of fire is eliminated.

• Due to lesser arc energy, air blast circuit breakers are very suitable for
conditions where frequent operation is required.

• The arcing products are completely removed by the blast whereas the oil
deteriorates with successive operations; the expense of regular oil is
replacement is avoided.

• The energy supplied for arc extinction is obtained from high pressure air
and is independent of the current to be interrupted.
• The arcing time is very small due to the rapid buildup of dielectric
strength between contacts. Therefore, the arc energy is only a fraction that in
oil circuit breakers, thus resulting in less burning of contacts.

Disadvantages:

• Considerable maintenance is required for the compressor plant which


supplies the air blast.

• Air blast circuit breakers are very sensitive to the variations in the rate
of restriking voltage.

• Air blast circuit breakers are finding wide applications in high voltage
installations.Majority of circuit breakers for voltages beyond 110 kV are of this
type.

Oil circuit breakers: Types, Construction, Operation,


Advantages, Disadvantages
Oil circuit breakers can be classified into following types: 1) Bulk oil circuit breakers 2) Low oil circuit
breakers,

Oil circuit breakers

Types Of Oil Circuit Breakers

Oil circuit breakers can be classified into following types:

1) Bulk oil circuit breakers


which use a large quantity of oil. In this circuit breaker the oil serves two
purposes. Firstly it extinguishes the arc during opening of contacts and
secondly it insulates the current conducting parts from one another and from
the earthed tank. Such circuit breakers are classified into:

• Plain oil circuit breakers

• Arc control circuit breakers

In the former type no means is available for controlling the arc and the contacts
are exposed to the whole of the oil in the tank. In the latter special arc control
devices are employed to get the beneficial action of the arc as efficiently as
possible

2) Low oil circuit breakers,

which use minimum amount of oil. In such circuit breakers oil is used only for
arc extinction, the current conducting parts are insulated by air or porcelain or
organic insulating material.

Construction

There are two chambers in a low oil circuit breaker; the oil in each chamber is
separated from each other. The main advantage of this is that low oil is required
and oil in second chamber won‘t get polluted. Upper chamber is called the
circuit breaker chamber and lower one is called the supporting chamber.
Circuit breaking chamber consists of moving contact and fixed contact.
Moving contact is connected with a piston it‘s just for the movement of the
contact and no pressure build due to its motion. There are two vents on fixed
contact they are axial vent for small current produced in oil due to heating of
arc and radial vents for large currents. The whole device is covered using
Bakelite paper and porcelain for protection. Vents are placed in a tabulator.

Operation

Under normal operating conditions, the moving contacts remain engaged with
the upper fixed contact. When a fault occurs, the moving contact is pulled down
by the tripping springs and an arc is struck. The arc vaporizes oil and produces
gases under high pressure. This action constrains the oil to pass through a
central hole in the moving contact and results in forcing series of oil through
the respective passages of the turbulator.The process of tabulation is orderly
one, in which the sections of arc are successively quenched by the effect of
separate streams of oil, moving across each section in turn and bearing away
its gases

constrains the oil to pass through a central hole in the moving contact and
results in forcing series of oil through the respective passages of the
turbulator.The process of tabulation is orderly one, in which the sections of arc
are successively quenched by the effect of separate streams of oil, moving
across each section in turn and bearing away its gases
Advantages
A low oil circuit breaker has following advantages compared to bulk oil circuit
breaker

• It requires lesser quantity of oil

• It requires smaller space

• There is reduced risk of fire

• Maintenance problems are reduced

Disadvantages

• Low oil circuit breaker has following disadvantages compared to bulk oil
circuit breaker

• Due to smaller quantity of oil, the degree of carbonization is increased

• There is a difficulty of removing the gases from the contact space in time

• The dielectric strength of oil deteriorates rapidly due to high degree of


carbonization.

• Prev Page

• Next P
SF6 circuit breaker: Construction, Operation,
Advantages, Disadvantages
At this point we are aware that the medium in which arc extinction of the circuit breaker takes place
greatly influences the important characteristics and life of the circuit breaker.

SF6 circuit breaker

At this point we are aware that the medium in which arc extinction of the circuit
breaker takes place greatly influences the important characteristics and life of
the circuit breaker. the working of a vacuum circuit breaker was illustrated. We
already know that the use of vacuum circuit breaker is mainly restricted to
system voltage below 38 kV. The characteristics of vacuum as medium and cost
of the vacuum CB does not makes it suitable for voltage exceeding 38 kV. In the
past for higher transmission voltage Oil Circuit Breaker (OCB) and Air Blast
Circuit Breaker (ABCB) were used. These days for higher transmission voltage
levels SF6 Circuit Breakers are largely used. OCB and ABCB have almost become
obsolete. In fact in many installations SF6 CB is used for lower voltages like 11
kV, 6 kV etc.. i)sulphur Hexafluoride symbolically written as SF6 is a gas which
satisfy the requirements of an ideal arc interrupting medium. So SF6 is
extensively used these days as an arc interrupting medium in circuit breakers
ranging from 3 kv upto 765 kv class. In addition to this SF6 is used in many
electrical equipments for insulation. Here first we discuss in brief, some of the
essential properties of SF6 which is the reason of it's extensive use in circuit
breakers

SF6 gas has high dielectric strength which is the most important quality of a
material for use in electrical equipments and in particular for breaker it is one
of the most desired properties. Moreover it has high Rate of Rise of dielectric
strength after arc extinction.

This characteristics is very much sought for a circuit breaker to avoid


restriking.
• SF6 is colour less, odour less and non toxic gas.

• SF6 is an inert gas. So in normal operating condition the metallic parts in


contact with the gas are not corroded. This ensures the life of the breaker and
reduces the need for maintenance.

• SF6 has high thermal conductivity which means the heat dissipation
capacity is more. This implies greater current carrying capacity when
surrounded by SF6 .

• The gas is quite stable. However it disintegrates to other fluorides of


Sulphur in the presence of arc. but after the extinction of the arc the SF6 gas is
reformed from the decomposition.

• SF6 being non-flammable so there is no risk of fire hazard and explosion.

A sulfur hexafluoride circuit breaker uses contacts surrounded by sulfur


hexafluoride gas to quench the arc. They are most often used for transmission-
level voltages and may be incorporated into compact gas-insulated switchgear.
In cold climates, supplemental heating or de-rating of the circuit breakers may
be required due to liquefaction of the SF6 gas.

Advantages:

• Due to superior arc quenching property of sf6 , such breakers have very
short arcing time

• Dielectric strength of sf6 gas is 2 to 3 times that of air, such breakers can
interrupt much larger currents.
• Gives noiseless operation due to its closed gas circuit

• Closed gas enclosure keeps the interior dry so that there is no moisture
problem

• There is no risk of fire as sf6 is non-inflammable

• There are no carbon deposits

• Low maintenance cost, light foundation requirements and minimum


auxiliary equipment

• sf6 breakers are totally enclosed and sealed from atmosphere, they are
particularly suitable where explosion hazard exists
Disadvantages:

• sf6 breakers are costly due to high cost of sf6

• sf6 gas has to be reconditioned after every operation of the breaker,


additional equipment is required for this purpose

CONSTRUCTION, PRINCIPLE OF OPERATION

The construction and working principles of SF6 circuit breaker varies from
manufacturer to manufacturer. In the past double pressure type of SF6
breakers were used. Now these are obsolete. Another type of SF6 breaker
design is the self blast type, which is usually used for medium transmission
voltage. The Puffer type SF6 breakers of single pressure type are the most
favored types prevalent in power industry. Here the working principle of Puffer
type breaker is illustrated (Fig-A)

As illustrated in the figure the breaker has a cylinder and piston arrangement.
Here the piston is fixed but the cylinder is movable. The cylinder is tied to the
moving contact so that for opening the breaker the cylinder along with the
moving contact moves away from the fixed contact (Fig-A(b)). But due to the
presence of fixed piston the SF6 gas inside the cylinder is compressed. The
compressed SFe gas flows through the nozzle and over the electric arc in axial
direction. Due to heat convection and radiation the arc radius reduces gradually
and the arc is finally extinguished at current zero.

The dielectric strength of the medium between the separated contacts


increases rapidly and restored quickly as fresh SF6 gas fills the space. While arc
quenching, small quantity of SF6 gas is broken down to some other fluorides of
sulphur which mostly recombine to form SF6 again. A filter is also suitably
placed in the interrupter to absorb the remaining decomposed byproduct.

The gas pressure inside the cylinder is maintained at around 5 kgf per sq. cm.
At higher pressure the dielectric strength of the gas increases. But at higher
pressure the SF6 gas liquify at higher temperature which is undesired. So
heater is required to be arranged for automatic control of the temperature for
circuit breakers where higher pressure is utilised. If the SF6 gas will liquify then
it loses the ability to quench the arc. Like vacuum breaker, SF6 breakers are
also available in modular design form so that two modules connected in series
can be used for higher voltage levels. SF6 breakers are available as both live
tank and dead tank types. In Fig-B above a live tank outdoor type 400 kV SF6
breaker is shown.

Vacuum circuit breakers: Construction, Operation,


Advantages, Disadvantages
In this breaker, vacuum is being used as the arc quenching medium. Vacuum offers highest insulating
strength, it has far superior arc quenching properties than any other medium.

vacuum circuit breakers

In this breaker, vacuum is being used as the arc quenching medium. Vacuum
offers highest insulating strength, it has far superior arc quenching properties
than any other medium. When contacts of a breaker are opened in vacuum, the
interruption occurs at first current zero with dielectric strength between the
contacts building up at a rate thousands of times that obtained with other
circuit breakers. Principle: When the contacts of the breaker are opened in
vacuum (10 -7 to 10 -5 torr), an arc is produced between the contacts by the
ionization of metal vapours of contacts. The arc is quickly extinguished because
the metallic vapours, electrons, and ions produced during arc condense quickly
on the surfaces of the circuit breaker contacts, resulting in quick recovery of
dielectric strength. As soon as the arc is produced in vacuum, it is quickly
extinguished due to the fast rate of recovery of dielectric strength in vacuum
Construction:

Fig shows the parts of a typical vacuum circuit breaker. It consists of fixed
contact, moving contact and arc shield mounted inside a vacuum chamber. The
movable member is connected to the control mechanism by stainless steel
bellows .This enables the permanent sealing of the vacuum chamber so as to
eliminate the possibility of leak. A glass vessel or ceramic vessel is used as the
outer insulating body. The arc shield prevents the deterioration of the internal
dielectric strength by preventing metallic vapours falling on the inside surface
of the outer insulating cover.
Working:

When the breaker operates the moving contacts separates from the fixed
contacts and an arc is struck between the contacts. The production of arc is due
to the ionization of metal ions and depends very much upon the material of
contacts. The arc is quickly extinguished because the metallic vapours,
electrons and ions produced during arc are diffused in short time and seized by
the surfaces of moving and fixed members and shields. Since vacuum has very
fast rate of recovery of dielectric strength, the arc extinction in a vacuum
breaker occurs with a short contact separation.
Advantages:

• They are compact, reliable and have longer life.

• There are no fire hazards

• There is no generation of gas during and after operation

• They can interrupt any fault current. The outstanding feature of a VCB is
that it can break any heavy fault current perfectly just before the contacts reach
the definite open position.
• They require little maintenance and are quiet in operation

• Can withstand lightning surges

• Low arc energy

• Low inertia and hence require smaller power for control mechanism.

Applications:

• For outdoor applications ranging from 22 kV to 66 kV. Suitable for


majority of applications in rural area.

Vacuum circuit breakers: Construction, Operation,


Advantages, Disadvantages
In this breaker, vacuum is being used as the arc quenching medium. Vacuum offers highest insulating
strength, it has far superior arc quenching properties than any other medium.

vacuum circuit breakers

In this breaker, vacuum is being used as the arc quenching medium. Vacuum
offers highest insulating strength, it has far superior arc quenching properties
than any other medium. When contacts of a breaker are opened in vacuum, the
interruption occurs at first current zero with dielectric strength between the
contacts building up at a rate thousands of times that obtained with other
circuit breakers. Principle: When the contacts of the breaker are opened in
vacuum (10 -7 to 10 -5 torr), an arc is produced between the contacts by the
ionization of metal vapours of contacts. The arc is quickly extinguished because
the metallic vapours, electrons, and ions produced during arc condense quickly
on the surfaces of the circuit breaker contacts, resulting in quick recovery of
dielectric strength. As soon as the arc is produced in vacuum, it is quickly
extinguished due to the fast rate of recovery of dielectric strength in vacuum

Construction:

Fig shows the parts of a typical vacuum circuit breaker. It consists of fixed
contact, moving contact and arc shield mounted inside a vacuum chamber. The
movable member is connected to the control mechanism by stainless steel
bellows .This enables the permanent sealing of the vacuum chamber so as to
eliminate the possibility of leak. A glass vessel or ceramic vessel is used as the
outer insulating body. The arc shield prevents the deterioration of the internal
dielectric strength by preventing metallic vapours falling on the inside surface
of the outer insulating cover.
Working:

When the breaker operates the moving contacts separates from the fixed
contacts and an arc is struck between the contacts. The production of arc is due
to the ionization of metal ions and depends very much upon the material of
contacts. The arc is quickly extinguished because the metallic vapours,
electrons and ions produced during arc are diffused in short time and seized by
the surfaces of moving and fixed members and shields. Since vacuum has very
fast rate of recovery of dielectric strength, the arc extinction in a vacuum
breaker occurs with a short contact separation.
Advantages:

• They are compact, reliable and have longer life.

• There are no fire hazards

• There is no generation of gas during and after operation

• They can interrupt any fault current. The outstanding feature of a VCB is
that it can break any heavy fault current perfectly just before the contacts reach
the definite open position.
• They require little maintenance and are quiet in operation

• Can withstand lightning surges

• Low arc energy

• Low inertia and hence require smaller power for control mechanism.

Applications:

• For outdoor applications ranging from 22 kV to 66 kV. Suitable for


majority of applications in rural area.

Electrical Drives
Motion control is required in large number of industrial and domestic applications like transportation
systems, rolling mills, paper machines, textile mills, machine tools, fans, pumps, robots, washing
machines etc.

Electrical Drives:

Motion control is required in large number of industrial and domestic


applications like transportation systems, rolling mills, paper machines, textile mills,
machine tools, fans, pumps, robots, washing machines etc.

Systems employed for motion control are called DRIVES, and may employ
any of prime movers such as diesel or petrol engines, gas or steam turbines, steam
engines, hydraulic motors and electric motors, for supplying mechanical energy for
motion control. Drives employing electric motors are known as Electrical Drives.
An Electric Drive can be defined as an electromechanical device for
converting electrical energy into mechanical energy to impart motion to different
machines and mechanisms for various kinds of process control.

Classification of Electric Drives

According to Mode of Operation

✓ Continuous duty drives


✓ Short time duty drives
✓ Intermittent duty drives

According to Means of Control

✓ Manual
✓ Semi-automatic
✓ Automatic

According to Number of machines

✓ Individual drive
✓ Group drive
✓ Multi-motor drive

According to Dynamics and Transients


✓ Uncontrolled transient period
✓ Controlled transient period

According to Methods of Speed Control

✓ Reversible and non-reversible uncontrolled constant speed.


✓ Reversible and non-reversible step speed control.
✓ Variable position control.

Reversible and non-reversible smooth speed control.

Advantages of Electrical Drive

They have flexible control characteristics. The steady state and dynamic
characteristics of electric drives can be shaped to satisfy the load requirements.

1. Drives can be provided with automatic fault detection systems.


Programmable logic controller and computers can be employed to
automatically control the drive operations in a desired sequence.
2. They are available in wide range of torque, speed and power.
3. They are adaptable to almost any operating conditions such as explosive and
radioactive environments
4. It can operate in all the four quadrants of speed-torque plane
5. They can be started instantly and can immediately be fully loaded
6. Control gear requirement for speed control, starting and braking is usually
simple and easy to operate.
Choice (or) Selection of Electrical Drives

Choice of an electric drive depends on a number of factors. Some of the important


factors are.

✓Steady State Operating conditions requirements:

Nature of speed torque characteristics, speed regulation, speed range,


efficiency, duty cycle, quadrants of operation, speed fluctuations if any, ratings etc

✓ Transient operation requirements:



Values of acceleration and deceleration, starting, braking and reversing
performance.

✓ Requirements related to the source:

Types of source and its capacity, magnitude of voltage, voltage fluctuations,


power factor, harmonics and their effect on other loads, ability to accept
regenerative power

✓ Capital and running cost, maintenance needs life.



✓ Space and weight restriction if any.

✓ Environment and location.

✓ Reliability.
Group Electric Drive

This drive consists of a single motor, which drives one or more line shafts
supported on bearings. The line shaft may be fitted with either pulleys and belts or
gears, by means of which a group of machines or mechanisms may be operated. It
is also sometimes called as SHAFT DRIVES.

Advantages

A single large motor can be used instead of number of small motors

Disadvantages

There is no flexibility. If the single motor used develops fault, the whole process
will be stopped.

Individual Electric Drive

In this drive each individual machine is driven by a separate motor. This


motor also imparts motion to various parts of the machine.

Multi Motor Electric Drive

In this drive system, there are several drives, each of which serves to actuate
one of the working parts of the drive mechanisms.
E.g. Complicated metal cutting machine tools

Paper making industries, rolling machines etc.

Classification of Electrical Drives

Another main classification of electric drive is

✓ DC drive
✓ AC drive

Applications

✓ Paper mills

✓ Cement Mills

✓ Textile mills

✓ Sugar Mills

✓ Steel Mills

✓ Electric Traction

✓ Petrochemical Industries

✓ Electrical Vehicles

Dynamics of Motor Load System


A motor generally drives a load (Machines) through some transmission system. While motor always
rotates, the load may rotate or undergo a translational motion.

Dynamics of Motor Load System

A motor generally drives a load (Machines) through some transmission


system. While motor always rotates, the load may rotate or undergo a
translational motion.

Load speed may be different from that of motor, and if the load has many
parts, their speed may be different and while some parts rotate others may go
through a translational motion.

Equivalent rotational system of motor and load is shown in the figure.

J = Moment of inertia of motor load system referred to the motor shaft kg / m 2

ωm = Instantaneous angular velocity of motor shaft, rad/sec.


T = Instantaneous value of developed motor torque, N-m

Tl = Instantaneous value of load torque, referred to the motor shaft N-m

Load torque includes friction and wind age torque of motor. Motor-load
system shown in figure can be described by the following fundamental torque
equation.

Equation (1) is applicable to variable inertia drives such as mine winders, reel
drives, Industrial robots.

For drives with constant inertia

Equation (2) shows that torque developed by motor

You might also like